Doch, natürlich gibt es den. Dass es einen Zeitpfeil gibt, ist unbestritten. (Auch wenn es nicht so klar ist, woher er kommt.) Zu jedem Zeitpunkt gibt es Ereignisse, die von diesem Zeitpunkt aus in der Vergangenheit liegen, und andere, die in der Zukunft sind. Welche das sind, ist an jedem Zeitpunkt anders. Ich versuche es mal wieder mit einer räumlichen Analogie: Stellt euch ein Seil vor, das von einem Haken herunterbaumelt. An jedem Punkt des Seils gibt es einen Teil des Seils, der oberhalb liegt und den gerade betrachteten Punkt (und alle darunter) trägt, und einen anderen Teil des Seils, dessen Last von diesem Punkt hier getragen (und nach oben weitergereicht) wird. Ein Punkt weiter oben hat viel Last zu tragen, einer weiter unten nur wenig. Die anliegende Last hat eine Richtung hat (nämlich in Richtung des Seils nach unten) und die rückhaltende Kraft hat ebenfalls eine Richtung (nach oben), so dass an jedem Punkt des Seils eine klare Zweiteilung zwischen lasttragendem Teil (Vergangenheit”) und lasterzeugendem Teil (“Zukunft”) herrscht.

Oder stellt euch als zweite (etwas umständliche) Analogie eine Straße vor, in der viele Häuser stehen. Es ist vollkommen finstere Nacht, aber dann biegt ein Auto in die Straße und beleuchtet die Häuser mit seinen Scheinwerfern.So etwa könnte das (schnell hingekritzelt) aussehen:

Wenn ich von meinem Standpunkt aus nach links in die Richtung des Autos schaue, dann sehe ich nur die unbeleuchteten Rückseiten der Häuser, habe also keine Ahnung, wie die Häuser dort aussehen. (Sie liegen in der “Zukunft”.) In der anderen Richtung dagegen kann ich die Häuser deutlich sehen. (Das ist die “Vergangenheit”.) Jemand in einem anderen Haus kann mehr oder weniger Häuser erkennen – trotzdem sind alle Häuser letztlich gleich, jedes steht nur an einem anderen Punkt in der Reihe.

Das Bild des Blockuniversums leugnet nicht, dass es einen Zeitpfeil gibt und dass die Welt für uns so aussieht, als würde die Zeit vergehen, genauso wenig wie eine Astronomin leugnet, dass es für uns so aussieht, als würde die Sonne sich um die Erde drehen. Die Astronomin erklärt uns ja nicht nur, warum dieses Bild falsch ist, sondern auch, warum es uns trotzdem so erscheint. Im Blockuniversum ist es genauso – weil es den Zeitpfeil gibt, trennen wir Ereignisse nach Vergangenheit und Zukunft und haben das Gefühl, dass sich die Grenze zwischen Vergangenheit und Zukunft verschiebt (so wie ihr in der Scheinwerferstraße das Gefühl hättet, dass sich die Grenze zwischen erleuchteten und dunklen Häusern verschiebt, wenn ihr die Straße entlang geht).

Kritiker des Blockuniversums führen gern als Gegenargument an, dass es mir dann ja egal sein müsse, ob ich einen unangenehmen Moment noch vor mir oder schon hinter mir habe. Aber das geht an der Sache vorbei, denn mein “jetziges” Ich hat den Termin eben “vor sich” und weiß, dass noch eine unangenehme Erfahrung bevorsteht. Unsere Gehirne sind so verdrahtet, dass wir in die Zukunft planen (denn den Zeitpfeil gibt es ja wirklich) und dass wir versuchen, unangenehme Dinge zu meiden und entsprechend zu handeln.  Auch Kritiker des Blockuniversums werden vermutlich zugeben, dass es eigentlich wirklich wenig hilfreich ist, mir heute Sorgen über etwas zu machen, das morgen eintritt und das ich nicht beeinflussen kann, aber unseren auf Planung konstruierten Gehirnen fällt es schwer, das umzusetzen.

Wo wir gerade bei Kritikern sind – die Wikipedia-Seite zum Thema führt als wichtigen Kritiker John Lucas an (der sich – nach meiner Ansicht – schon in Sachen Gödelscher Satz mit wenig argumentativem Ruhm bekleckert hat (Wer mag, kann ja auf seine Wikipedia-Seite gehen und mal schauen, wie schnell er eine Widerlegung für die “anthropic mechanical thesis” findet), aber das ist eine andere Geschichte…). Lucas sagt

the Block universe gives a deeply inadequate view of time. It fails to account for the passage of time, the pre-eminence of the present, the directedness of time and the difference between the future and the past

1 / 2 / 3 / 4

Kommentare (142)

  1. #1 Quacki
    19. September 2012

    “Wir sind jetzt im jetzt!”

    (Ich hab leider keinen besseren Link gefunden, aber die Szene ist auch in Gänze voll lustig!)

  2. #2 Quacki
    19. September 2012

    Eine Freundin von mir hat mal erzählt, dass sie mit ein paar Leuten am Diskutieren war. Jemand kam vorbei und fragte, worum es ging. “Ob es einen freien Willen gibt.” “Ist doch egal!” war die prompte Antwort. Will sagen: Wenn ich das richtig verstanden habe, dann ist das Blockuniversum doch lediglich ein Bild, wie man sich das Vergehen der Zeit vorstellen kann, oder nicht? (Ein hübsches Bild, wie ich finde: Das Universum quasi in Scheiben hintereinandergelegt 🙂 Ein extradimensionales Wesen könnte dann in der Zeit wie in einem Buch blättern …). Hat die Sichtweise irgendwelche Konsequenzen, d.h. gibt es eine mögliche Messung, um festzustellen dass das Blockuniversum das richtige Bild ist und kein anderes?

  3. #3 Alderamin
    19. September 2012

    @Martin B

    Ich hatte es ja nebenan (17. September) schon gesagt, aber hier passt es besser:

    Laut Brian Greene scheint die Zeit deswegen zu fließen, weil es eine Richtung der Entropiezunahme gibt, d.h. der Zunahme der Unordnung (und dahinter verbirgt sich auch die Ursache der Kausalität). Da Energiegewinnung und damit auch der menschliche Metabolismus und unser Gehirn auf der Zunahme von Entropie beruhen, muss unsere Wahrnehmung zwangsläufig der Entropierichtung folgen und somit nehmen wir diese als Zeitrichtung wahr, so Greene sinngemäß. Dann braucht es keinen Dualismus.

    Die Entropierichtung ist wiederum eine einfache Wahrscheinlichkeitssache. Ein hohes Maß an Ordnung ist stets unwahrscheinlicher als ein gewisses Maß an Unordnung. Wenn man einen perfekt ausgerichteten Zauberwürfel (Rubik’s Cube, kennt den überhaupt noch jemand?) hat und fünf mal zufällig daran dreht, dann hat man ein wildes Durcheinander. Und zwar eigentlich immer, egal, wie man dreht. Wenn man aber das Chaos einmal angerichtet hat, kommt man nicht einfach durch fünf mal zufälliges Drehen zurück, man muss exakt die richtigen fünf Drehungen in der richtigen Reihenfolge vollführen, was zufällig höchst unwahrscheinlich wäre. Es gibt also viele Wege in’s Chaos und wenige hinaus. Deswegen nimmt das Chaos im Mittel stets zu.

    Greene erklärt dann, dass das Universum in einem höchst geordneten Zustand begonnen hat (man könnte auch sagen, das Blockuniversum hat an einem Ende einen höchst sortierten Zustand, oder man könnte sagen, aus dem Anfangszustand führen sehr viele Übergänge hinaus und nur wenige zurück). Nur deswegen ist es möglich, eine Zeitrichtung vorzugeben.

    Greenes Buch “Der Stoff aus dem der Kosmos ist” ist bei Astrodicticum simplex ausführlich rezensiert, siehe dort Linkliste rechts. Ich muss sagen, nach der Lektüre hatte ich zum ersten Mal den Eindruck, verstanden zu haben, was Zeit eigentlich ist.

    Die Frage ist allerdings noch, wie Zufälle und zahlreiche Möglichkeiten im Blockuniversum Platz finden. Ist alles vorbestimmt und es gibt nur einen einzigen Zeitablauf oder spaltet sich das Universum ständig in neue Zustände? Zufällige Ereignisse in der Quantenwelt sollen ja nach Experimenten keine versteckten Parameter haben, die wir nur nicht überblicken, sondern wirklich zufällig sein. Ist das noch das Modell des Blockuniversums, oder schon eine Viele-Welten-Theorie? Im Prinzip reichten zur Modellierung viele Zustände, aber nur eine Welt, die wie ein Rinnsal durch permanent zufällige Richtungsänderungen zwischen den Steinen ihren Weg nach unten, in Richtung steigender Entropie, sucht.

  4. #4 Aiko
    Hier oder Dort
    19. September 2012

    Interessant – Musik ist ein viel schöneres und geeigneteres Beispiel für das Zeit-Bewusstsein, das immer subjektiv ist. Edmund Husserl hat hier den Begriff der Retention eingeführt und die Fähigkeit des Bewusstseins damit beschrieben, vergangene Eindrücke zu vergegenwärtigen und damit Noten eines Musikstückes zusammen zusetzen zu einem Ganzen.
    Objektiv gibt es den Ton und die Töne, also diese Jetzt(e) – subjektiv gibt es die Tonfolge, der Verlauf, der durch die Wahrnehmung auch als Gestalt gesehen wird. Und das ist eben eine Leistung des Bewusstseins, des Hörbewusstseins. Ähnlich ist es beim Film, der ja auch Einzelbilder aneinander reiht – .
    Cage spielt dann z.B. in seiner Komposition ORGAN²/ASLSP mit diesem Zeitbewusstsein. Nächster Impuls/Ton am 5. Oktober 2013.

  5. #5 Erik Ihle
    19. September 2012

    Zeit fliesst nicht, weil wir Bewegungen betrachten und deren Eigenschaften wahrnehmen. Es fliessen also die Veränderungen von Eigenschaften ( Energiezustände ). Warum verändert sich im Blockuniversum überhaupt etwas? Ein Grund ist, das sich Energie in einen Zustand “Materie” und einen weiteren Zustand “Antimaterie” wandelt, formt, wie auch immer. Dabei Antimaterie einen vollständigen Ablauf (von Anfang bis zu seinem Ende) sehr schnell abwickelt und Materie seinen vollständigen Ablauf sehr langsam abwickelt. Beide Zustände sind durch die Ordnung der Quantelung von Energie und deren Verschränkung miteinander verbunden. 
    Zeit wie wir sie im Alltag wahrnehmen beruht auf Bewegungen, die ihre Quelle aus der Verschränkung zwischen Antimaterie und Materie bezieht. Darum ist Zeit eine Illusion, Bewegung aber nicht.

    Zeit als Blockuniversum betrachtet vergeht nicht, ist aber in seinem Wirken für alle Bewegungen im Raum verantwortlich. Zeit ist in diesem Sinne auch nicht mehr nur 4. Dimension, sondern trägt in sich auch Sinn und Ziel. Die “Substanz” Zeit, wie ich sie nenne, soll alle Bewegung von Energie “vergehen” lassen.

    Ein Text von Erik dem Unlesbaren.

  6. #6 rolak
    20. September 2012

    Wenn ich mich recht entsinne, bgann ich das mit der Zeit etwas anders zu sehen, als mir endlich aufging, wie diese komischen Zielphotos zu interpretieren sind. Im Gegensatz zu sonst – räümliche Übersicht zu einem Zeitpunkt – ist DA eine zeitliche Übersicht an einem Raumpunkt zu sehen.

    Schöner Text zum verlinken.

  7. #7 Aiko
    20. September 2012

    Die Planck-Zeit beschreibt das kleinstmögliche Zeitintervall, für das die bekannten Gesetze der Physik gültig sind. Sie ergibt sich aus der Zeit, die Licht benötigt, um eine Planck-Länge zurückzulegen und eine (gedachte) Zustandsveränderung zu bewirken.
    Bei kleineren Zeitintervallen verliert die Zeit ihre vertrauten Eigenschaften als Kontinuum. Sie würde quantisieren, d. h. Zeit liefe unterhalb der Planck-Zeit in diskreten Sprüngen ab. Aus den bekannten physikalischen Gesetzen folgt, dass jedes Objekt, das einen Vorgang kürzer als in Planck-Zeit durchlebt, zu einer Singularität wird.
    (aus wiki)

  8. #8 AP
    20. September 2012

    Ich finde derartige Gedanken über die Zeit und ihren Ablauf faszinierend.

    Die vielen “Jetzt” in deiner zweiten Grafik, die gleichwertig “nebeneinander” liegen, sind also so etwas wie kleinstmögliche Momente? Entsprechen die Abstände zwischen diesen Momenten dann der Planck-Zeit? Und würde das nicht bedeuten, dass der Zeitpfeil nicht kontinuierlich verliefe, sondern von Planck-Einheit zu Planck-Einheit springt, also quasi die Änderung der Entropiezustände der Einheiten “ausliest”?

  9. #9 MartinB
    20. September 2012

    @Quacki
    Direkt messbare Konsequenzen sehe ich nicht, aber eine konsistente Beschreibung ist im Blockuniversum gerade wegen der Probleme mit RT und QM sicher einfacher.

    @Alderamin
    Ja, das ist doch im wesentlichen genau, was ich sage: Den Zeitpfeil (ob der wirklich einfach auf der Entropie beruht ist nicht so klar, siehe den link zum Penrose-Artikel) gibt es unabhängig davon, ob die Zeit “vergeht”

    @Aiko
    Stimmt, mit Musik gibt es auch schöne Analogien – bei Hofstadter gibt es die Szene, wo der Krebs eine Schallplatte als Kunstwerk ansieht und durch draufschauen quasi das Musikstück als Ganzes erfassen kann.

    @AP
    Nein, die Scheiben sind einfach nur eine grafische Darstellung, die Zeit darf trotzdem kontinuierlich sein, ich kann nur überabzählbar unendlich viele Pfeile so schlecht malen.

  10. #10 Alderamin
    20. September 2012

    @AP

    Die vielen “Jetzt” in deiner zweiten Grafik, die gleichwertig “nebeneinander” liegen, sind also so etwas wie kleinstmögliche Momente?

    Ich glaube nicht, dass es einen globalen Taktgeber (oder ein Pixelmuster) gibt, der das ganze Weltall in ein einheitliches Raster unterteilt. Ein bewegter Beobachter hat ja auch ein dazu “schräges” Raster, für ihn verlaufen die “jetzt”-Linien in einem Winkel zu denen eines ruhenden Beobachters.

    Und würde das nicht bedeuten, dass der Zeitpfeil nicht kontinuierlich verliefe, sondern von Planck-Einheit zu Planck-Einheit springt, also quasi die Änderung der Entropiezustände der Einheiten “ausliest”?

    Die Idee beim Blockuniversum ist ja, dass da gar nichts springt, sondern einfach “ist”. Was sich verändert ist im wesentlichen die Anordnung der Objekte, die sich bewegen.

    Zeitlich engstmöglich benachbarte Zustände würden sich aber mit dem von Aiko gesagten darin unterscheiden, dass ein Lichtstrahl (und nichts ist schneller) maximal eine Plancklänge weiter fortgeschritten sein kann, alles langsamere hätte sich hingegen noch gar nicht verändern können. Wenn es eine Plancklänge gibt, macht das Sinn, aber so viel ich weiß wurde eine Quantelung des Raumes bisher noch nicht nachgewiesen.

  11. #11 Erik Ihle
    20. September 2012

    GleichZeitigkeit (1.) und Quantelung (2.) von Energie (3.) mit einer Verschränkung (4.) in alle Zeit-Räume / Bewegungsräume (5.) sind schwer vorstellbar. Sie vollständig zu beschreiben ist viel schwerer. – Stimmts Herr MartinB?

    Danke für Ihren Artikel, denn er hat mir diese Vorstellung ermöglicht.

  12. #12 Erik Ihle
    20. September 2012

    @Alderamin
    “… ein Lichtstrahl (und nichts ist schneller) …!
    In einem gequanteltem Energieraum, innerhalb des Blockuniversums, ist diese Aussage richtig.
    In einem Informations-Universum mit seiner Gleichzeitigkeit durch die Verschränkung von Quanten, gilt obige Aussage nicht mehr. Gleichzeitigkeit an Information ist das schnellste was ich mir vorstellen kann.
    -> gerade habe ich eine Eingebung, das es auch noch schneller geht. So ist das mit mir, dem Troll und dem, der unlesbare Texte schreibt. Was geht nur im Verstand und (m)einem Bewusstsein so vor?
    Viele liebe Grüsse . . . ..

  13. #13 AndreasM
    20. September 2012

    Vergehen und Fluss sind hier ja konsequent in Anführungszeichen gesetzt. Das sollte wohl bewusst machen, dass sie hier gar nicht sauber definiert werden, am Anfang zumindest.
    Der Artikel ist weit mehr philosphisch als naturwissenschaftlich.

    Subjektiv existiert für uns ein Zeitfluss, der tatsächlich unterschiedliche Geschwindigkeiten haben kann, deren Einheit Sekunden pro Moment ist. Mit Moment ist hier der Wahrnehmungsmoment gemeint, der sich aus unserer Informationsverarbeitung im Hirn ergibt (und der Speicherung, da wir uns häufig erst später über den Zeitfluss gedanken machen).

    Objektiv ist ein Zeitfluss wohl nur, wie später im Artikel, an Ursache und Wirkung festzumachen. Und diese Abhängigkeiten werden ja naturwissenschaftlich untersucht. Verbleibt die Frage, wie ich das dann darstelle. Es ist aber rein philosophisch, eine dieser Darstellungen als richtiger als die anderen zu sehen.

  14. #14 MartinB
    20. September 2012

    @Andreas
    “Der Artikel ist weit mehr philosphisch als naturwissenschaftlich.”
    Ja, ist er. Steht ja auch drin. Ist das irgendwie problematisch?

    “Es ist aber rein philosophisch, eine dieser Darstellungen als richtiger als die anderen zu sehen.”
    Aber naturwissenschaftlich gibt es eben kein gutes Argument dafür, ein “Vergehen” der Zeit anzunehmen, darum geht es ja.

  15. #15 Alderamin
    20. September 2012

    @MartinB

    Ja, das ist doch im wesentlichen genau, was ich sage: Den Zeitpfeil (ob der wirklich einfach auf der Entropie beruht ist nicht so klar, siehe den link zum Penrose-Artikel) gibt es unabhängig davon, ob die Zeit “vergeht”

    Ich widerspreche Dir ja auch nicht, ich hatte die Entropiegeschichte deswegen hier erwähnt, weil Du oben im Text schriebst:

    Wenn ihr euch auf den Standpunkt eines Dualisten stellt und annehmt, dass es eine Seele gibt, die außerhalb unserer Welt steht und euren Körper “bewohnt”, dann könnt ihr sinnvoll vom “Vergehen der Zeit” sprechen – so wie oben im Beispiel mit dem Buch. Wer will, mag das als Argument für den Dualismus nehmen. (Ich sehe das anders – auch die Idee, dass wir ein Bewusstsein haben, dass einem Körper innewohnt, ist meiner Ansicht nach eine Fiktion, aber das ist wieder ein anderes Thema und sicher bei Gelegenheit einen anderen Text aus der Reihe “Ein Physiker versucht sich als Philosoph” wert…)

    Womit Du am Ende nicht gesagt hast, wie Du Dir das scheinbare Vergehen der Zeit erklärst. Greene erklärt es halt mit der Entropiezunahme, die auch die Übergänge in unserem Gehirn bestimmt. Dem würde ich mich anschließen.

    Vermutlich/anscheinend habt Ihr das beim Penrose-Artikel schon durchgekaut, auf den ich erst später gestoßen bin. Die ganzen Kommentare zu diesem Artikel muss ich erst noch nacharbeiten, da waren ja wohl mal einige eine Zeit verschütt gegangen, wenn ich das richtig verstanden habe. Sind die wieder alle da (bis auf den Umstellungstag)?

  16. #16 Frank Wappler
    https://woerterbuchnetz.de/DWB/?sigle=DWB&mode=Vernetzung&lemid=GD00935
    20. September 2012

    Martin Bäker schrieb (September 19, 2012):
    > […] So ist es auch mit der Zeit: […]

    Und im ganzen Text findet sich keine Spur von “Dauer” …

    Gar keine?

    Nein! —
    einen klitzekleinen Anklang an diesen wichtigen Begriff hat sich der genannte Autor (vielleicht in einem Augenblick des Abgelenktseins) doch geleistet:

    > […] ihr könnt beispielsweise die Reise von Frodo Beutlin vom Auenland nach Mordor lesen, die ziemlich genau ein halbes Jahr gedauert hat. [Hervorhebung FW]

    p.s.

    > In der Relativitätstheorie ist das Konzept der “Gegenwart” wesentlich problematischer als in der Newtonschen Physik

    Was könnte denn problematischer sein, als Begriffe gänzlich ohne Zugrundelegung einer nachvollziehbaren Messdefinition zu benutzen; etwa wie den Begriff “Gegenwart (mehrerer verschiedener Beteiligter” in der Newtonschen Physik ?

    > weil die Frage, ob zwei Ereignisse gleichzeitig sind, (also “dieselbe Gegenwart” haben) vom Beobachter abhängt.

    Die (rhetorische) Frage, ob “zwei Ereignisse gleichzeitig” wären, stellt ja offensichtlich einen Kategoriefehler dar.
    Sinnvoller Weise fragt man ja stattdessen, ob “zwei Zeiten gleichzeitig” wären, oder nicht;
    bzw. in Einsteins Worten, ob “eine bestimmte Zeigerstellung des einen Beobachters und eine bestimmte Zeigerstellung des anderen Beobachters (einander) gleichzeitig” wären, oder nicht.

  17. #17 nihil jie
    20. September 2012

    ich spiele schon seit längerem mit dem Gedanken, dass das Vergehen der Zeit überhaupt möglich ist, weil es eine höchste Geschwindigkeit im Universum gibt, mit der sich Informationen überhaupt übertragen und ausbreiten können. Gäbe sie es diese Begrenzung nicht, gäbe es keine Kausalität, denn dann könnte wirklich alles Gleichzeitig passieren. Dann würde womöglich nicht mal das Bild eines Blockuniversums Sinn ergeben. Es gäbe dann vielleicht ein Universum in dem es nur das Jetzt gibt.

    Gut… ich könnte jetzt etwas komplett Falsch interpretieren und daher zu dem Bild kommen. Aber naja… ich schreibe keine publikationen von daher kann ich mir solche “Gedanken” auch leisten ohne mich komplett lächerlich zu machen *gg 😉

  18. #18 MartinB
    20. September 2012

    @Alderamin
    “Womit Du am Ende nicht gesagt hast, wie Du Dir das scheinbare Vergehen der Zeit erklärst.”
    Nein, ich habe nur gesagt, dass es einen Zeitpfeil gibt und dieser sicherlich in der Physik begründet ist – ob der 2. Hauptsatz der Thermodynamik als Begründung ausreicht, ist aber nicht so klar, siehe auch den verlinkten Artikel zum Penrose-Buch.

    @Nihil je
    Das mit der begrenzten Geschwindigkeit ist ein guter Gedanke, aber man kann sich ja problemlos ein echtes Newton-Universum vorstellen – haben die Menschen ja auch lange genug gemacht.
    Ich finde es immer extrem gefährlich, wenn man durch angeblich “bloßes Nachdenken” Dinge ableitet, die erst die moderne Physik entdeckt hat, das macht mich imemr argwöhnisch…

  19. #19 mar o
    20. September 2012

    Was mir noch nicht klar ist, ist warum die Aussage “Zeit fließt nicht” zum Blockuniversum führt. Das liegt aber wahrscheinlichauch daran, dass mir aus dem Artikel gar nicht klar wurde, was das Blockuniversum ausmacht. Was sind denn z.B. die Blöcke?

    In der klassischen Mechanik ist die Zeit einfach ein Parameter, da fließt also erst mal auch nichts. Rede ich jetzt schon über das Blockuniversum, wenn ich mir Observablen als 6N+1-dimensionale Objekte vorstelle anstelle von 6N-dimensionalen, die sich zeitlich entwickeln?

    Außerdem sehe ich auch das Problem mit der Quantenmechanik. Wie bastele ich den Zufall ins Blockuniversum? In der Kopenhagener Interpretation liegt die Zukunft ja eben noch nicht fest.

  20. #20 MartinB
    20. September 2012

    @Mar o
    Der Begriff “Block” bezieht sichnur darauf, dass man die einzelnen Zeit-“Scheiben” sozusagen zu einem Block stapelt und quasi “von außen” als ganzes betrachtet.
    “In der klassischen Mechanik ist die Zeit einfach ein Parameter, da fließt also erst mal auch nichts.”
    Sage ich ja auch – das Bild des Blockuniversums ergibt auch in der klassichen Physik Sinn.
    “Wie bastele ich den Zufall ins Blockuniversum? ”
    Da sehe ich kein Problem – entlang des zeitpfeils geguckt kann man aus dem Zustand zur Zeit t den Zustand zur Zeit t+dt nicht eindeutig vorhersagen, das wäre dann “Zufall”.

  21. #21 mar o
    20. September 2012

    Da sehe ich kein Problem – entlang des zeitpfeils geguckt kann man aus dem Zustand zur Zeit t den Zustand zur Zeit t+dt nicht eindeutig vorhersagen, das wäre dann “Zufall”.

    Das kann ich nur aus dem lokalen Standpunkt heraus nachvollziehen. Wenn die Entwicklung der Welt von außen betrachtet einem Stapel Blätter mit Zustandsbeschreibungen gleicht, dann muss der lokal empfundene Zufall emergent sein. Und damit schränke ich mich auf deterministische Interpretationen der Quantenmechanik ein.

  22. #22 MartinB
    20. September 2012

    @mar o
    Das halte ich für kein schlüssiges Argument. Wenn ich jetzt ein Photon durch einen passenden Polarisator schicke, dann ist es Zufall, ob es durchkommt. Ist es morgen – rückwirkend geguckt – kein Zufall mehr, sondern deterministisch geworden?
    Der Begriff “zufall” braucht einen Zeitpfeil, aber den gibt es ja. Ein Ereignis ist zufällig, wenn aus es dem aktuellen Zustand (in der unmittelbaren Vergangenheit) nicht vorhergesagt werden kann. Das gilt auch im Blockuniversum.

  23. #23 HF
    20. September 2012

    Interessante Anregungen.
    Das Blockuniversum passt gut zum Determinismus. Jeder einzelne Zeitschnitt enthält die gesamte Imformation über Vergangenheit und Zukunft, die ganze Struktur ist starr. Mit einem Billardkugeluniversum gibt es kein Problem. Schon die Thermodynamik bringt aber ein leichtes Unbehagen ( siehe obige Posts ). Der Mikrozustand gehört zum Billardkugeluniversum, der Gleichgewichts (Makro)zustand ist zeitlich unveränderlich und durch wenige Bits charakterisiert. Wie beschreibt man aber Systeme, die große Speicher enthalten oder aus speichernden Bauelementen konstruiert sind? Ein Gehirn ist so ein System, aber ich habe den Eindruck, dass die Physik mit solchen Systemen nichts rechtes anzufangen weiss. Jedes Atom zu beschreiben ist viel zu detailliert, Volumen, Druck und Temperatur sind sicherlich zu wenig.

  24. #24 mar o
    20. September 2012

    Das ist natürlich ein guter Punkt. Da muss ich auf jeden Fall mal ein bisschen drüber nachdenken.

  25. #25 Frank Wappler
    https://universe.et
    20. September 2012

    Martin Bäker schrieb (#18, September 19, 2012):
    > […] man kann sich ja problemlos ein echtes Newton-Universum vorstellen […]

    Wie käme man denn zu einem nachvollziehbaren Begriff von “Distanz”-Verhälnissen (oder zumindest Verhältnissen von “Quasi-Distanz”),
    oder zu einem nachvollziehbaren Begriff von “Geschwindigkeit”,
    in “einem echten Newton-Universum” ?

    also ohne Zugrundelegung der entsprechenden Messdefinitionen der RT und Betrachtung von geeigneten Näherungen (insbesondere [tex] \beta \ll 1 [/tex] bzw. [tex] lim \beta \rightarrow 0 [/tex])

    ?
    &#1F631; ??

  26. #26 MartinB
    20. September 2012

    @HF
    “Jeder einzelne Zeitschnitt enthält die gesamte Imformation über Vergangenheit und Zukunft, die ganze Struktur ist starr.”
    Das ist – wie schon gesagt – nicht notwendigerweise so, auch ein Blockuniversum kann Zufall enthalten (im oben definierten Sinn).

    Was die Beschreibung komplexer Systeme angeht, macht die Physik in meine Augen genau das richtige: Sie liefert Grundlagen an die Chemie, die wiederum an die Biochemie usw., so dass jede Disziplin die jeweils emergenten Strukturen anguckt und sich um die tieferen Ebenen nicht kümmern muss.

  27. #27 Heino
    20. September 2012

    # auch die Idee, dass wir ein Bewusstsein haben, dass einem Körper innewohnt, ist meiner Ansicht nach eine Fiktion,#

    da bin ich mal gespannt, wo du das erklärst. Hab nämlich große Schwierigkeiten mir vorzustellen, wer der Träger einer solchen Fiktion sein soll.

  28. #28 MartinB
    20. September 2012

    @Heino
    Gemeint ist, dass es keinen Sinn ergibt, ein Bewusstsein einem Körper “innewohnen” zu lassen, so wie wir auch das Konzept einer “Lebenskraft” nicht mehr verwenden, die der Materie innewohnt und sie lebendig macht. Dein Bewusstsein ist ein Phänomen, das durch den Körper erzeugt wird, genauso wie z.B. “Bewegung”.
    Aber es wird noch etwas dauern, bis ich dazu was schreibe (zumal ich im Moment eine fiese Erkältung habe und mein Hirn nur auf Sparflamme läuft).

  29. #29 Heino
    20. September 2012

    dass es “irgendwie” keinen Sinn ergibt mit der Lebenskraft, verstehe ich schon. Ich hab auch kein Problem irgend etwas, was zur Selbststeuerung eines Organismus da ist, als Bewusstsein zu bezeichnen. Bloß die ICH-Erfahrung, die erklärt sich so nicht.

    Mach mal was gegen deine Erkältung und dass dein Hirn wieder läuft! Ich bin neugierig 😉

  30. #30 HF
    20. September 2012

    “auch ein Blockuniversum kann Zufall enthalten”
    Schon richtig, aber das Bild verliert dann, zumindest für mich, viel von seiner Faszination.
    “… macht die Physik in meine Augen genau das richtige”
    Ob das Paar ( Reduktion/Emergenz ) gelungen ist oder sinnvoll ist, muss sich auch an der Nützlichkeit für den Konstrukteur messen lassen. Der Biochemiker kann auf seiner Beschreibungsebene nützliches tun, aber auch jederzeit auf z.B. Berechnungsmodelle der Quantenchemie zurückgreifen. Es ist genau diese Verzahnung, die die Einheit der Wissenschaften ausmacht. Können informationsverarbeitende Systeme Gegenstand der Physik sein, oder gehören sie in eine andere Beschreibungsebene?

  31. #31 MartinB
    20. September 2012

    @Heino
    Ich glaube, das Problem steckt darin, was man als Erklärung ansehen mag. Lass uns das lieber vertagen, bis ich weiß, was ich genau schreiben will (eigentlich wird die Zielrichtung wohl eine etwas andere sein) und bis ich wieder entklebte Neuronen habe.

  32. #32 HF
    20. September 2012

    Kann man mit der Vorstellung eines Blockuniversums die Funktion von ( digitalem ) Speicher beschreiben?
    Als Gegenentwurf kann man auch die Existenz der Vergangenheit anzweifeln, es gibt nichts als die Gegenwart und die Aufzeichnungen, die auf die Vergangenheit hinweisen. Die Zukunft existiert nur in den Köpfen der Menschen, als unvollkommene Simulation unter Zuhilfenahme des Gedächtnisses.

  33. #33 Heino
    20. September 2012

    Hallo Martin,

    gute Besserung.

  34. #34 MartinB
    20. September 2012

    @HF
    “Können informationsverarbeitende Systeme Gegenstand der Physik sein”
    Fällt nur bedingt in unser Ressort, weil bei den Info-verarbeitenden Systemen die Hardwarebasis ja meist zweitrangig ist.

    “Als Gegenentwurf kann man auch die Existenz der Vergangenheit anzweifeln, es gibt nichts als die Gegenwart und die Aufzeichnungen, die auf die Vergangenheit hinweisen.”
    Kann man vielleicht machen (ist das nicht ein Konzept bei den hinduistischen Religionen, nach dem die Welt in jedem Moment neu geschaffen und wieder zerstört wird?) – ich finde es aber unbefriedigend, weil es dann wieder die Frage aufwirft, was den den “aktuellen” Moment auszeichnet, den es als einzigen geben soll. Wenn es jeden Moment “als einzigen” gibt, dann bin ich doch wieder beim Blockuniversum.
    Das mit dem digitalen Speicher habe ich nicht verstanden.

  35. #35 Ludger
    20. September 2012

    Wenn ich Esoteriker wäre, würde ich behaupten, dass die Gegenwart ein Phasenübergang des Aggregationszustandes der Raumzeit sei: Die Raumzeit der Vergangenheit sei quasi fest, die Gegenwart quasi flüssig und die Zukunft quasi gasförmig. Da man mit Analogien jeden Blödsinn begründen kann, hat diese Aussage aber wohl keinen Wert. So, nach dieser Anmerkung habe ich nun eine Frage: Gibt es eine denkbare Situation / Konstellation, in der sich der Zeitpfeil umkehrt und was hätte das zu bedeuten? ( Ich habe mal so etwas in einem SciFi-Buch gelesen, in dem es um ein Überleben des Big Crunch ging. )

  36. #36 MartinB
    20. September 2012

    @Ludger
    “Gibt es eine denkbare Situation / Konstellation, in der sich der Zeitpfeil umkehrt und was hätte das zu bedeuten? ”
    Es gab mal das Argument, dass bei einem Big Crunch das Universum ja im Volumen schrumpft und deswegen die Entropie wieder abnehmen sollte. Das halten aber meines Wissens die wenigsten für korrekt. Ist nett erklärt bei Penrose im Buch “Emperors New Mind” (toll in Sachen Physik, gruselig in Sachen Gödel und Bewusstsein).

  37. #37 StefanL
    20. September 2012

    Man könnte auch “Zeit, das unbekannte Wesen” betiteln.
    Relativ aktuell zu zwei Zeit-dimensionen https://arxiv.org/abs/1001.2485
    und au contraire Scientists suggest spacetime has no time dimension

    Noch eine Frage zur Quantenverschränkung: da wird ja immer gesagt wenn A den Zustand a mißt dann würde dadurch bestimmt, daß B eben b mißt/beobachtet – diese scheinbar kausale Beziehung hebt sich doch auf wenn beide gleichzeitig messen -oder?

  38. #38 Stefan W.
    20. September 2012

    Wenn die Zeit vergehen könnte, dann könnte sie irgendwann vergangen sein, und nicht weiter existieren. Das wäre von begrenzter Lustigkeit.

    Das Vergehen (der Zeit der Beatles, des Tages, der Jugend) selbst ist die Zeit. Zeit und Vergehen sind wesensgleich.

    Mann kann auch eine Zeit mit einer anderen messen – etwa ‘das Fussballspiel dauerte 94 Minuten’.

    Dass es einen Zeitpfeil gibt, ist unbestritten.

    Naja. Was soll das denn sein – der Zeitpfeil? Eine Idee doch wohl. Wenn man eine Idee hatte, dann wäre es albern zu leugnen, dass es die Idee gibt. Was soll also gesagt werden?

    Das “Fließen” der Zeit ist letztlich nur eine Illusion.

    Man kann sich ohne Rückgriff auf die Zeit schlichtweg kein Fließen vorstellen – auch kein Fahren, Gehen, keine Bewegung, keine Änderung. All diesen Phänomenen ist die Idee einer Zeit inhärent, eine vorher-nachher Relation.

    Wenn man vom konkreten Phänomen absieht – das Toastbrot schimmelt, das Wasser fließt die Wupper runter, die Marzipankartoffeln erobern die Supermarktregale zurück – und Altern oder Fliessen abstrakt bezeichnen will, dann sagt man die Zeit verginge, aber das ist nicht so zu verstehen, als ob die Zeit altert und dann mal bald tot vom Stuhl kippt oder in der Sonne zerbröselt.

    Aber vielleicht habe ich auch das Problem nicht verstanden.

  39. #39 Fossilium
    21. September 2012

    Hi Martin,
    das Fliessen der Zeit beweist, dass es uns garnicht gibt -denn wir leben in einer fiktiven Welt. Beweis:
    In der Z u k u n f t extistieren wir noch nicht, die Zukunft beherbergt ja die zukünftige und nicht die gegenwärtige Existenz – in der V e r g a n g e n h e it existieren wir nicht, weil wir ja da schon vergangen sind – also existieren wir nur in der Gegenwart. Die aber ist eine unendluch dünne Schnittstelle zwischen Zukunft und Vergangenheit – und in einer unendlich dünnen Schnittstelle kann logischerweise nichts Endliches existieren.
    Also wo ist hier der Denkfehler – oder gibt es den garnicht ?
    Grüsse Fossilium

  40. #40 Stefan W.
    21. September 2012

    Der Denkfehler ist, die Schnittstelle mit “unendlich dünn” als räumlich darzustellen, um sie in einen Widerpruch mit einer endlichen Existenz zu bringen, ohne zu sagen, wieso das logischerweise folgert. Woraus folgert das logisch?

    Wenn die Existenz von etwas in einer unendlich kurzen Zeitspanne unmöglich wäre, dann müsste man unendlich kurze Zeitspannen ablehnen – nicht Existenzen, die man ja als Beobachtet voraussetzt.

  41. #41 MartinB
    21. September 2012

    @StefanW
    Der Zeitpfeil drückt aus, dass es eine Asymmetrie zwischen Vergangenheit und Zukunft gibt, analog zum Seil.
    Ich argumentiere hier ja nicht, dass es keine Zeit gibt, sondern nur, dass die Zeit nicht “vergeht”.

  42. #42 MartinB
    21. September 2012

    @StefanL
    Genau, die kausale Beziehung funktioniert nicht, wenn die beiden Messungen raumartig getrennt sind, also so, dass zwischen den Messereignissen kein Lichtsignal ausgetauscht werden kann.

    Den Artikel, nach dem die Zeit nur eine Ordnung im Raum sein soll, habe ich gar nicht verstanden.

    @Fossilium
    Nicht böse sein, aber solche schein-logischen Argumente helfen in meinen Augen wenig, weil da immer irgendwelche Annahmen drin stecken, z.B. die, dass die Zeit kontinuierlich ist oder dass in einer unendlich dünnen Schnittstelle nichts existieren kann (warum nicht – theoretisch könnte unser Universum ja auch eine unendlich dünne Scheibe in einem höherdimensionalen Raum sein?).

  43. #43 AP
    21. September 2012

    Ich lese interessiert alle Kommentare und freue mich über die Vielzahl an Ideen. Doch eines lässt mich nicht los:
    Wie erklärt sich die Diskrepanz zwischen “die Zeit schreitet nicht voran” und “es gibt einen Zeitpfeil”?
    Wenn die Zeit aus aneinandergereihten, gleichwertigen Augenblicken besteht, und der Zeitpfeil lediglich das Ergebnis der Entropiezunahme darstellt, wie erklärt sich dann das subjektive Empfinden des Vergehens der Zeit, des Alterns, der (objektiven) Erosion und Evolution etc., also des offensichtlichen Fortschreitens der Zeit?

    PS: Es gibt offenbar keine Benachrichtigungsfunktion mehr, wenn neue Kommentare geschrieben worden sind. Oder doch?

  44. #44 MartinB
    21. September 2012

    @AP
    Das subjektive Empfinden erklärt sich dadurch, dass wir zu jedem Zeitpunkt eine Erinnerung an die Vergangenheit haben, aber nicht an die Zukunft. Das sollte das Haus-Beispiel verdeutlichen.

  45. #45 Erik Ihle
    https://www.facebook.com/erik.ihle?ref=tn_tnmn
    21. September 2012

    @AndreasM
    Wie kann naturwissenschaftlich dargestellt werden, das Zeit nicht vergeht und einen Raum wie das Blockuniversum bilden kann? Leonard Euler hat fünf mathematische Konstanten in einer Gleichung zusammen gefügt. Ich behaupte, das die Eulersche Identität obige Fragestellung beantworten kann.
    Auch kann diese Gleichung die Vorstellung entwickeln, wie sich Higgsteilchen und Blockuniversum “verbinden” (miteinander kommunizieren). Diese Gleichung kann noch vieles mehr . . . ..

    Ich lade Herrn Martin Bäkker und Herrn Andreas Müller vom Exzellenzcluster Universe der TU München ein, ein Bild zu den Grundlagen (Eulersche Identität) eines Informations-Universums zu formulieren. Gerne möchte ich Herrn Anton Zeilinger später in diesen Arbeitsprozess einbinden.
    Vorher sollten die Forschungsgelder für die “2. Exzellenzrunde” des Universe Cluster ihren Nutzen beweisen und eine Grundstruktur der Ordnung eines Austausches von Informationen im Universum öffentlich darstellen.

    Freitag, der 21. September 2012 – das ist mein Tag, den gönne ich mir heute . . . ..

  46. #46 StefanL
    21. September 2012

    Besser(?) beschrieben ist diese Idee von ‘time is a numerical order of motion’ hier https://www.vixra.org/pdf/1106.0026v1.pdf .
    So wie ich das verstehe ist da der Grundgedanke, daß wir uns in einem 4-dimensionalem Minkowski-Raum (nicht Raumzeit) bewegen und Zeit eine Art Effekt darin ist aber keine eigenständige Dimension. Allerdings bezieht sich der/die Autor/en dann doch wieder auf eine elementare Planckzeit.
    Und es wird – zumindest mir – auch nicht wirklich klar ob da “Zeit” dann als dimensionsloser Skalar (numerical ordering) oder als ein emergentes Phänomen (perception of motion/change) zu betrachten ist .

  47. #47 MartinB
    21. September 2012

    @StefanL
    Was ist denn ein 4D-Minkowski-Raum? Entweder es ist euklidisch, oder es ist kein reiner Raum, oder nicht? Jedenfalls verstehe ich schon die ersten zwei Seiten des papers nicht (was an meinem Gesundheitszustand liegen mag…)

  48. #48 Alderamin
    21. September 2012

    @AP

    Wenn die Zeit aus aneinandergereihten, gleichwertigen Augenblicken besteht, und der Zeitpfeil lediglich das Ergebnis der Entropiezunahme darstellt, wie erklärt sich dann das subjektive Empfinden des Vergehens der Zeit, des Alterns, der (objektiven) Erosion und Evolution etc., also des offensichtlichen Fortschreitens der Zeit?

    Eigentlich ganz einfach: Wahrnehmung und Empfindung ist Verarbeitung im Gehirn. Verarbeitung im Gehirn ist Zustandsänderung im Gehirn. Zustandsänderung ist Übergang und Übergang ist Zeitpfeil. Die Denkprozesse beruhen auf Reaktionen, die Energie verbrauchen (= in Wärme umwandeln = Entropiezunahme), deswegen laufen sie in in Richtung zunehmender Entropie ab. Oder anders ausgedrückt, ein Verarbeitungsergebnis kann nur in derjenigen Richtung des Blockuniversums liegen, in der die Entropie insgesamt größer ist als in der entgegengesetzten Richtung.

  49. #49 StefanL
    21. September 2012

    Lol – eigentlich sollten solche Fragen ja den Autoren des Papers gestellt werden 😉 . Die naheliegende Interpretation ist aber wohl 4-dim Euklidischer Raum/Mannigfaltigkeit mit üblichem Skalarprodukt (pg. 2) – sonst hätten die Autoren tatsächlich eine spezifische Bilinearform mitliefern sollen. Die spezifische Minkowski Metrik wird ja nur als spezielle Lösung einer Lagrangschen Multiplikatorengleichung gennannt. Das sie bei der Bezeichung “Minkowski-Raum” blieben ist möglicherweise auch dem Umstand geschuldet damit zum Ausdruck zu bringen, daß sich an den uns normal erscheinenden drei Raumdimensionen nichts ändert. Sie sprechen ja auch von “Minkowskian arena” (pg.2 und die Verweise auf dieses “toy model” ( und bzgl. Deinem Gesundheitszustand: Gute Besserung!)

  50. #50 AndreasM
    21. September 2012

    “Aber naturwissenschaftlich gibt es eben kein gutes Argument dafür, ein “Vergehen” der Zeit anzunehmen, darum geht es ja.”
    “Ich argumentiere hier ja nicht, dass es keine Zeit gibt, sondern nur, dass die Zeit nicht “vergeht”.”
    @MartinB: Was ist denn naturwissenschaftlich gesehen das Vergehen oder Fliessen von Zeit? Denn nur wenn man das im naturwissenschaftlichen Kontext definiert, ist es auch eine These, dass Zeit nicht vergeht.
    Vergehen und Fliessen sind Begriffe, die Änderung über ein Bezugssystem wiedergeben. Bei Verwendung im naturwissenschaftlichen Rahmen ist das normalerweise die Zeit. In der subjektiven Anwendung auf die Zeit ist das Bezugssystem die Wahrnehmung und Erinnerung. Aber was ist das objektive Bezugssystem, wenn man die Begriffe auf die Zeit selbst anwenden will. Da fällt mir erstmal nur die Kausalität ein.
    Fliesst die Zeit, wenn es eine Kausalordnung gibt?

  51. #51 sabine puttins
    korschenbroich
    21. September 2012

    Salve, wehrte Gelehrte der Experimental- und Teilchenphysik!

    WOW – das geballte Wissen des Studienganges Physik bis zum Vordiplom… meines Erachtens fehlen da nur Schrödingers Katzen, Entropien und die etwas detailliertere Verschwurbelung der Chaostheorie… Oder sollte ich in der Eile ob der Menge vll. diese exakte Aufdröselung überlesen haben? Allerdings fehlt mir der “humane Faktor der Metaphysik” ein wenig, wesgleich ich mich nun darum bemühe;-=)

    Und so kann ich als humaner Kalfaktor zumindest die Meta-Ebene touchieren, da ich die Zeit persönlich, wie in meinem Blog dargestellt “– innehaltend ob der temporalen Verquickung verzückt durch die Allmacht der Zeit” während meines längerem Komas als eine Funktion einer digital gerasterten Matrix wahrnahm, die meine Vision vom Leben darstellte.

    Diese digital gerasterte Matrix stellte meine Visionen von Zeit und Raum dar – abzüglich des temporalen Elements der Zeit, die in Form eines Nahrungsschlauches durch meinen Körper floss.

    So wurde meine “Koma-Welt” zu einer abstrahierten Essenz meiner Realität in Entitäten, deren Verifikation ich – abzüglich des temporalen Elements – in Form der Nahrungssonde, in theoretischer Form abstrakt “nachschwingen” musste.

    Da ich aufgrund meines Geburtsdatum zum Datum 24/7 und den daraus aus astrologischen und ähnlichen aus Meta-Schwafel resultierenden Prädiktionen die Zeit schlechthin darstelle,

    (siehe:https://frankfutt.de/2012/04/21/cerebrale-konstrukta/)

    und mich seitdem ich denke, aktiv mit dem temporalen Entschleunigen befasst habe, darf ich temporal und räumlich losgelöst existieren, denn bei mir in meiner Chaos-Bude (Foto davon auch auf Blog:

    https://frankfutt.de/2012/04/06/mietminderung-berechtigt-vermieter-verbietet-nutzung-von-wohnraumen/)

    ist nichts zur gleichen Zeit an derselben Stelle vorzufinden, per definitionem herrscht also die stetige (sich steigernde) Entropie…

    In diesem zeitlich entkernten Sinne verbleibe ich mit den Qualen des Leids des Wartens auf – hoffentlich temporal tangierende – Antwort(en) mit
    Grüßen
    Sabine

  52. #53 Niels
    21. September 2012

    @StefanL @MartinB
    “Physics Essays” ist wohl so etwas wie “Chaos, Solitons & Fractals”, nur viel kleiner und unbekannter.
    Das kommt eben davon, wenn man die Cranks dazu auffordert, ihre “Theorien” zuerst mal in einer wissenschaftlichen Zeitschrift zu veröffentlichen. Dann gründen sie einfach selbst “wissenschaftliche” Journale und machen das wirklich. 😉

    Mal ein paar Hintergrundinformationen, die ich in fünf Minuten ergoogelt habe:
    Zwei der drei Autoren sind die einzigen beiden “Forscher” des “Space Life Institute”.
    spacelife.si
    Siehe dort auch die “Individual research method EHAN” und die Bibliographien der beiden Autoren. Reicht meiner Meinung schon, wenn man die Namen der Journale und die Titel der dort von diesen Autoren veröffentlichten Arbeiten liest.
    z.B.
    “Basic frequency of a-temporal physical space as a driving force of evolution.”
    “Conscious communication into politics.”
    “A-temporal physical space and introduction to the theory of everything.”
    “Integration of science and religion with self-experience of the observer.”

    Einer der beiden ist übrigens ein “independent scholar”, der auch ein Buch veröffentlicht hat.
    Einstein’s Timeless Universe: The Foundation for Cosmic Religiousness
    https://www.amazon.com/Einsteins-Timeless-Universe-Foundation-Religiousness/dp/3843375739/ref=cm_cr_pr_product_top

    Jedenfalls verstehe ich schon die ersten zwei Seiten des papers nicht

    Das ist auch unmöglich.
    Dort wird versucht, dieses Paper
    https://arxiv.org/abs/0903.4876
    nachzuerzählen, obwohl man es nicht verstanden hat. Dann wird so getan, als ob das dort vorgestellte toy-model unser Universum beschreiben würde. Was es natürlich nicht tut.

    Ist absolute Zeitverschwendung, sich damit zu beschäftigen.

    Das “Original-Paper”
    https://arxiv.org/pdf/0903.4876v1.pdf
    kann man mal überfliegen, muss man aber auch nicht.
    Da muss noch extrem viel Arbeit hineinfließen, bevor es auch nur grob abschätzbar wird, ob dieser Ansatz überhaupt zu irgend etwas führen kann.
    Und wenn man sich alle auf arxiv veröffentlichten Ansätze anschauen will, hat man echt was zu tun.

  53. #54 Niels
    21. September 2012

    Jetzt hab ich extra die “https://www” weggelassen, damit ich nicht im Spam-Filter lande.
    Jetzt sind sie wieder drin und deswegen flieg ich trotzdem raus?

  54. #55 AP
    21. September 2012

    @Alderamin:

    Soweit okay, aber das setzt einen Beobachter, ein Gehirn voraus, welches diese Wahrnehmungen macht. Du gehst aber nicht auf meine Beispiele Erosion und Evolution ein. Denn selbst wenn es keine empfindenden Wesen aka Menschen gäbe, würde Erosion und Evolution stattfinden.

    Nehmen wir die Evolution auf einem hypothetischen Planeten ohne empfindungsfähigen, intelligenten, höheren Lebewesen, also nur Pflanzen ohne Beobachter. Was bestimmt dort den Zeitpfeil? Außerdem scheint mir, dass Evolution, welches ja im Allgemeinen eine Erhöhung der Komplexität eines Lebewesens bedeutet, eine Abminderung der Entropie bewirkte. Entropie würde kleiner. Und doch schreitet die Evolution mit dem Zeitpfeil voran. Was ist in einem solchen Fall mit dem Zeitpfeil? Was ist der Wirkmechanismus?

  55. #56 MartinB
    21. September 2012

    @Niels
    Tja, die Freuden der Filter. Immerhin sehe ich im neuen System sofort, wenn was im Filter hängt, beim alten musste man immer erst extra gucken.

    Danke für den Hinweis zu “Physics Essays” – ich habe mich schon gewundert, warum das bei vixra liegt und nicht bei arxiv…

    @AndreasM
    “Vergehen und Fliessen sind Begriffe, die Änderung über ein Bezugssystem wiedergeben. ”
    Genau. Und mein Argument (soweit ich das mit den beiden funktionsfähigen Neuronen, die ich im Moment habe, nachvollziehen kann…) ist ja, dass man, damit Zeit “vergehen” kann, ein weiteres Bezugssystem bräuchte, damit man von “außen” draufgucken kann.

    @AP
    Der Zeitpfeil kann physikalisch mehrere Ursachen haben, entweder die Entropiezunahme oder auch den QM-Messprozess, der ja auch irreversibel ist. Wird ganz gut bei Penrose (s.o.) diskutiert.

    @Sabine
    Mag an meiner fiesen Erkältung liegen, aber ich verstehe kein Wort. (Hinweis: “Matrix” ist zur zeit das Modewort, für das man vor ein paar Jahrzehnten “Struktur” sagte und noch früher “Seiendes” o.ä. – sagt erst mal wenig bis gar nichts aus…)

  56. #57 StefanL
    21. September 2012

    @Niels – danke für die Aufklärung über “Physics Essays”. D.h. dann wohl auch das da kein (brauchbarer) peer review stattfindet (?). Darüber gestolpert bin ich ja von phys.org/news her kommend. Wie kann/sollte man das i.a. einordnen?… und zu den suspekten inhaltlichen Teilen ist ja auch schon einiges angemerkt.

  57. #58 Alderamin
    21. September 2012

    @AP

    Soweit okay, aber das setzt einen Beobachter, ein Gehirn voraus, welches diese Wahrnehmungen macht. Du gehst aber nicht auf meine Beispiele Erosion und Evolution ein. Denn selbst wenn es keine empfindenden Wesen aka Menschen gäbe, würde Erosion und Evolution stattfinden.

    Zeit ist ja Veränderung. Physikalische Prozesse bedeuten, dass sich etwas bewegt oder ein Teilchen entsteht oder vergeht und dergleichen. Das heißt, benachbarte Zeitscheiben im Blockuniversum unterscheiden sich (übrigens genau so wie benachbarte Raumscheiben im 3D-Raum, ohne dass da etwas fließt). Das Problem ist nur, dass alle physikalischen Reaktionen vorwärts wie rückwärts ablaufen können, so dass es auf den ersten Blick keinen Grund zu geben scheint, warum die Zeit in einer bestimmten Richtung laufen soll, und nicht in der Gegenrichtung. Der Witz ist aber, dass eben eine gegebene Ordnung durch Veränderung praktisch irreversibel zerstört wird – das ist das Entropieargument. Wenn man mit einer hohen Ordnung anfängt, kann es nur bergab gehen. Darum zeigt der Zeitpfeil nur in eine Richtung.

    Erosion ist letztlich nichts anderes, als Entropiezunahme. Ein gut sortierter Berg zerlegt sich in Geröll, das den Berg hinunter rollt. Es gibt nur eine Konfiguration für den konkreten Berg, aber unzählige Möglichkeiten, diesen klein zu krümeln. Das ist Entropiezunahme.

    Evolution ist ein bisschen komplizierter, die verbraucht Energie. Unter Energiezufuhr kann die Entropie lokal auch abnehmen, z.B. kann man Gas in eine Druckflasche pumpen oder einen Kühlschrank innen kälter als die Umgebung machen. Bei der dazu nötigen Energieerzeugung wird aber wieder mehr Entropie erzeugt, als bei der durch sie verrichteten Arbeit gewonnen werden kann, deswegen stimmt die Bilanz am Ende.

    Also, Erosion und Evolution (inklusive der nötigen Energieerzeugung) “passieren” – genau wie Denkprozesse im Hirn – in Richtung der zunehmenden Entropie. “Passieren” heißt einfach, der Zustand ändert sich. Es passiert auch etwas, wenn man dem Zeitpfeil entgegen schaut, die Zeitscheiben in der falschen Sortierung betrachtet, aber das Hirn schaut aufgrund seiner Funktionsweise mit dem Zeitpfeil und sieht den Berg zerkrümeln und nicht wachsen. Ohne Wahrnehmung sind benachbarte Zeitscheiben einfach unterschiedlich und die Naturgesetze bestimmen, wie die nächste Scheibe aus der vorherigen hervorgeht. Dadurch entsteht eine Richtung mit zunehmender Entropie, die Gegenrichtung hat abnehmende.

    Ungefähr klar?

    @Martin

    Ich habe alle Kommentare vom Penrose-Artikel durch, aber die entscheidenden scheinen noch zu fehlen. Vom Messprozess habe ich nichts gelesen, daher hier nur die Entropie-Erklärung, so wie ich sie verstanden habe.

  58. #59 MartinB
    22. September 2012

    @Alderamin
    In dem penrose-Artikel ist das der Absatz, der mit
    “Hier sieht Penrose ein Problem in der gegenwärtigen Physik”
    beginnt, da steckt das Messproblem drin.
    Wenn der Messprozess ein “echtes” physikalisches Phänomen ist, dann ist er auch irreversibel.

    Das andere, was du geschrieben hast, hat mich auf eine andere Idee zur Veranschaulichung gebracht: Wenn man die einzelnen “Scheiben”, die ich oben gemalt habe, wie Spielkarten einfach “mischen” würde, dann würde das an der Wahrnehmung einer Person zu einem bestimmten Zeitpunkt nichts ändern – man würde die aktuelle Welt sehen und daraus auf die Vergangenheit (kleinere Entropie) schließen. Die gezeigte Reihenfolge der Karten ist nichts zusätzliches, sondern ist die natürliche Reihenfolge, die die Karten bereits mit sich bringen (die Entropie liefert quasi die Nummerierung, wenn man mal die Entropie als alleinverantwortlich für den Zeitpfeil annimmt).

  59. #60 Alderamin
    22. September 2012

    @MartinB

    Danke für den Hinweis auf den Absatz im Penrose-Artikel.

    Zu dem zweiten Punkt: im Prinzip ja, aber total mischen macht physikalisch keinen Sinn, es muss ja eine Kontinuität gegeben sein, die von der Physik vorgeschrieben wird, wenn man zur Nachbarkarte wechselt. Z.B. eine kontinuierliche Bewegung oder eine Teilchenreaktion. Es gibt sozusagen eine Reihenfolge zwischen den Karten, die von den physikalischen Gesetzen vorgeschrieben wird.

    Der Witz ist jetzt aber, dass es im allgemeinen zwei Richtungen für jeden Vorgang gibt: jede Teilchenreaktion, jede Newtonsche Bewegung kann auch anders herum laufen. Deswegen ist es physikalisch nicht vollkommen widersinnig, die Karten auch anders herum anzuordnen. Aber warum sollte in irgendeiner Richtung die Entropie abnehmen?

    Greene argumentiert in “Der Stoff aus dem der Kosmos ist” folgendermaßen: wenn man den Zeitpfeil mal ignoriert, dann gibt es eigentlich gar keinen Grund, warum irgendein Übergang in welcher Richtung auch immer zu einem unwahrscheinlicheren Zustand führen sollte. Auch die Vergangenheit müsste eigentlich eine höhere Entropie haben. Ich zitiere mal eine entscheidende Stelle (S. 191, 2. Absatz):

    Ein häufiges Missverständnis beruht auf der Annahme, dass die Entropie, da sie dem Zweiten Hauptsatz der Thermodynamik zufolge in Richtung Zukunft zunimmt, in Richtung Vergangenheit zwangsläufig abnehmen müsse. […] Tatsächlich besagt der Zweite Hauptsatz nämlich: wenn ein physikalisches System zu einem gegebenen Zeitpunkt und zu einem früheren Zeitpunkt nicht zufällig seine maximale Entropie besitzt, ist es außerordentlich wahrscheinlich, dass dieses System zu einem späteren Zeitpunkt und zu einem früheren Zeitpunkt höhere Entropie gehabt hat beziehungsweise haben wird. […] Bei Gesetzen, die für eine Unterscheidung zwischen Vergangenheit und Zukunft blind sind, ist eine solche Zeitsymmetrie unvermeidlich.

    Und später auf Seite 196, wo er beschreibt, dass man um 22:30 ein Glas Wasser mit teilweise geschmolzenen Eiswürfeln vorfindet und dann die Vorgeschichte rekonstruieren soll:

    Aus Sicht der Wahrscheinlichkeit ist es absurd, diesen niederentropischen Zustand durch Berufung auf einen Zustand noch geringerer Entropie, noch geringerer Wahrscheinlichkeit erklären zu wollen, nämlich die Behauptung, es wären noch geordnetere, noch vollständigger ausgeformte Eiswürfel in einer früheren, geordneteren Umgebung entdeckt worden. Vielmehr spricht die Wahrscheinlichkeit mit überwältigendem Nachdruck dafür, dass die Dinge in einem wenig überraschenden, vollkommen normalen, hochentropischen Zustand begannen: mit einem Glas voll gleichmäßig flüssigem Wasser ohne eine Spur von Eis. Durch eine zwar unwahrscheinliche, aber gelegentlich doch zu erwartende statistische Fluktuation verstieß das Glas Wasser gegen den Zweiten Hauptsatz der Thermodynamik und entwickelte sich zu einem Zustand geringerer Entropie, in dem teilweise geformte Eiswürfel entstanden.

    […weiter auf S. 198…]

    Daher gilt, was auf die Bar zutrifft, in umso höherem Maße für das gesamte Universum: Es ist weit wahrscheinlicher – unvorstellbar viel wahrscheinlicher -, dass das gesamte Universum, das wir heute vor Augen haben, als statistisch seltene Fluktuation aus einer normalen, wenig überraschenden, hochentropischen und vollkommen ungeordneten Konfiguration entstanden ist. […] Und, was Blotzmannns Erkenntnis angeht, wenn das Universum lange genug wartet – vielleicht fast eine Ewigkeit -, wird sein üblicher, hochentropischer, hochwahrscheinlicher, vollkommen ungeordneter Zustand durch das eigene Gestoße, Gedränge und seine Zufallsströme von Teilchen und Strahlung über kurz oder lang die Konfiguration annehmen, die wir genau jetzt erblicken. Unsere Körper, unsere Gehirne erwüchsen vollkommen ausgeformt aus dem Chaos – versehen mit Erinnerungen, Kenntnissen, Fertigkeiten -, obwohl die Vergangenheit, die sie wiederzuspiegeln scheinen, in Wirklichkeit nie stattgefunden hat.”

    Das muss man sich mal vor Augen führen (man sollte das Buch lesen, dann kann man dem Gedankengang besser folgen, weil er doch ziemlich heftig ist). Es ist wahrscheinlicher, dass das Universum gerade eben erst mit all unseren Erinnerungen aus dem Nichts entstanden ist, als aus einem Zustand niedrigerer Entropie, der noch viel unwahrscheinlicher entstehen würde.

    Greene rettet die Logik dann damit, dass der extrem niederentropische Zustand, der zu Beginn des Universums geherrscht hat, von der kosmischen Inflation hervorgebracht wurde (S. 365):

    Ich bin von dieser Geschichte der inflationären Kosmologie und des Zeitpfeils hingerissen. In einer ungezügelten und hochenergetischen Region des Ur-Chaos entstand durch eine ultramikroskopische Fluktuation ein gleichförmiges Inflaton-Feld, das nicht einmal das Gewichtslimit für Handgepäck überschritt. Dadurch wurde eine inflationäre Expansion ausgelöst, die dem Zeitpfeil eine Richtung gab und der Rest ist Geschichte.

    Damit sind wir jetzt wieder bei Penrose.

  60. #61 Alderamin
    22. September 2012

    Sorry für den Doppelpost, aber da war eine blockquote-Klammer offen geblieben, daher zu Lesbarkeit nochmal:

    @MartinB

    Danke für den Hinweis auf den Absatz im Penrose-Artikel.

    Zu dem zweiten Punkt: im Prinzip ja, aber total mischen macht physikalisch keinen Sinn, es muss ja eine Kontinuität gegeben sein, die von der Physik vorgeschrieben wird, wenn man zur Nachbarkarte wechselt. Z.B. eine kontinuierliche Bewegung oder eine Teilchenreaktion. Es gibt sozusagen eine Reihenfolge zwischen den Karten, die von den physikalischen Gesetzen vorgeschrieben wird.

    Der Witz ist jetzt aber, dass es im allgemeinen zwei Richtungen für jeden Vorgang gibt: jede Teilchenreaktion, jede Newtonsche Bewegung kann auch anders herum laufen. Deswegen ist es physikalisch nicht vollkommen widersinnig, die Karten auch anders herum anzuordnen. Aber warum sollte in irgendeiner Richtung die Entropie abnehmen?

    Greene argumentiert in “Der Stoff aus dem der Kosmos ist” folgendermaßen: wenn man den Zeitpfeil mal ignoriert, dann gibt es eigentlich gar keinen Grund, warum irgendein Übergang in welcher Richtung auch immer zu einem unwahrscheinlicheren Zustand führen sollte. Auch die Vergangenheit müsste eigentlich eine höhere Entropie haben. Ich zitiere mal eine entscheidende Stelle (S. 191, 2. Absatz):

    Ein häufiges Missverständnis beruht auf der Annahme, dass die Entropie, da sie dem Zweiten Hauptsatz der Thermodynamik zufolge in Richtung Zukunft zunimmt, in Richtung Vergangenheit zwangsläufig abnehmen müsse. […] Tatsächlich besagt der Zweite Hauptsatz nämlich: wenn ein physikalisches System zu einem gegebenen Zeitpunkt und zu einem früheren Zeitpunkt nicht zufällig seine maximale Entropie besitzt, ist es außerordentlich wahrscheinlich, dass dieses System zu einem späteren Zeitpunkt und zu einem früheren Zeitpunkt höhere Entropie gehabt hat beziehungsweise haben wird. […] Bei Gesetzen, die für eine Unterscheidung zwischen Vergangenheit und Zukunft blind sind, ist eine solche Zeitsymmetrie unvermeidlich.

    Und später auf Seite 196, wo er beschreibt, dass man um 22:30 ein Glas Wasser mit teilweise geschmolzenen Eiswürfeln vorfindet und dann die Vorgeschichte rekonstruieren soll:

    Aus Sicht der Wahrscheinlichkeit ist es absurd, diesen niederentropischen Zustand durch Berufung auf einen Zustand noch geringerer Entropie, noch geringerer Wahrscheinlichkeit erklären zu wollen, nämlich die Behauptung, es wären noch geordnetere, noch vollständigger ausgeformte Eiswürfel in einer früheren, geordneteren Umgebung entdeckt worden. Vielmehr spricht die Wahrscheinlichkeit mit überwältigendem Nachdruck dafür, dass die Dinge in einem wenig überraschenden, vollkommen normalen, hochentropischen Zustand begannen: mit einem Glas voll gleichmäßig flüssigem Wasser ohne eine Spur von Eis. Durch eine zwar unwahrscheinliche, aber gelegentlich doch zu erwartende statistische Fluktuation verstieß das Glas Wasser gegen den Zweiten Hauptsatz der Thermodynamik und entwickelte sich zu einem Zustand geringerer Entropie, in dem teilweise geformte Eiswürfel entstanden.

    […weiter auf S. 198…]

    Daher gilt, was auf die Bar zutrifft, in umso höherem Maße für das gesamte Universum: Es ist weit wahrscheinlicher – unvorstellbar viel wahrscheinlicher -, dass das gesamte Universum, das wir heute vor Augen haben, als statistisch seltene Fluktuation aus einer normalen, wenig überraschenden, hochentropischen und vollkommen ungeordneten Konfiguration entstanden ist. […] Und, was Blotzmannns Erkenntnis angeht, wenn das Universum lange genug wartet – vielleicht fast eine Ewigkeit -, wird sein üblicher, hochentropischer, hochwahrscheinlicher, vollkommen ungeordneter Zustand durch das eigene Gestoße, Gedränge und seine Zufallsströme von Teilchen und Strahlung über kurz oder lang die Konfiguration annehmen, die wir genau jetzt erblicken. Unsere Körper, unsere Gehirne erwüchsen vollkommen ausgeformt aus dem Chaos – versehen mit Erinnerungen, Kenntnissen, Fertigkeiten -, obwohl die Vergangenheit, die sie wiederzuspiegeln scheinen, in Wirklichkeit nie stattgefunden hat.

    Das muss man sich mal vor Augen führen (man sollte das Buch lesen, dann kann man dem Gedankengang besser folgen, weil er doch ziemlich heftig ist). Es ist wahrscheinlicher, dass das Universum gerade eben erst mit all unseren Erinnerungen aus dem Nichts entstanden ist, als aus einem Zustand niedrigerer Entropie, der noch viel unwahrscheinlicher entstehen würde.

    Greene rettet die Logik dann damit, dass der extrem niederentropische Zustand, der zu Beginn des Universums geherrscht hat, von der kosmischen Inflation hervorgebracht wurde (S. 365):

    Ich bin von dieser Geschichte der inflationären Kosmologie und des Zeitpfeils hingerissen. In einer ungezügelten und hochenergetischen Region des Ur-Chaos entstand durch eine ultramikroskopische Fluktuation ein gleichförmiges Inflaton-Feld, das nicht einmal das Gewichtslimit für Handgepäck überschritt. Dadurch wurde eine inflationäre Expansion ausgelöst, die dem Zeitpfeil eine Richtung gab und der Rest ist Geschichte.

    Damit sind wir jetzt wieder bei Penrose.

  61. #62 AP
    22. September 2012

    Ich habe nun auch den Pensoreartikel gelesen und die Kommentare. Ich habe auch verstanden, warum selbst die Evolution die Entropie erhöht. – Wenn also jeglicher “natürlicher” Prozess die Entropie erhöht, dann gibt es auch den Zeitpfeil . Soweit okay.

    Was mich aber noch beschäftigt ist Folgendes:
    Wenn man nun in die Zukunft schaut, sagen wir 100 Mrd. Jahre oder so, wenn sämtliche Materie im Universum in SL gelangt ist und diese SL wieder sauber durch Hakwing-Strahlung vollkommen verdampft sind, dann kann sich die Entropie ja nicht mehr erhöhen, weil das gesamte Universum in einem Zustand homogener Strahlung existiert. Es kann keine Zunahme mehr an Unordnung geben. Heißt das, dass dann der Zeitpfeil endet? Oder gibt es dann noch QM-Messprozesse im Strahlungsozean, die den Zeitpfeil aufrecht erhalten?

    Zweite Frage:
    Wie sieht es mit dem Zeitpfeil bei virtuellen Teilchenpaaren aus (QF). Entropie kann sich dabei ja nicht erhöhen (oder seh ich das falsch?). Lässt sich in diesem Fall der Zeitpfeil überhaupt festlegen oder könnte er genauso gut in die Gegenrichtung zeigen. Wie ist in diesem Fall das “Scheiben”-Prinzip anzuwenden?

  62. #63 Alderamin
    22. September 2012

    @AP

    Die Antworten auf Deine Fragen stehen eigentlich alle in den Stellen drin, die ich aus dem Greene-Buch zitiert habe. Entropie ist eine Wahrscheinlichkeits-Geschichte, es kann lokal durch statistische Schwankungen auch eine niedrigere Entropie entstehen, wenn man nur lange genug wartet (die Chance, dass sich sämtliche Luftmoleküle in Deinem Zimmer in eine Ecke zurückziehen, ist real gegeben, man muss nur lange genug warten).

    Und bei Teilchenreaktionen, virtuell oder nicht, ist kein Zeitpfeil gegeben. Der entsteht erst durch die Statistiik, wenn viele Reaktionen betrachtet werden, z.B. was nötig ist, um eine Tasse auf dem Boden zerspringen zu lassen oder sie umgekehrt wieder zusammen zu setzen.

  63. #64 MartinB
    22. September 2012

    @AP
    Die Idee, dass irgendwann sozusagen alles im Maximum-Entropie-Hitzetod endet, ist ja schon recht alt. Aber wenn wir tatsächlich ein nur von thermischer Strahlung erfülltes Universum haben könnten, dann gäbe es keinen Zeitpfeil – es sei denn, wir berücksichtigen, dass sich das Universum ja trotzdem ausdehnt, das ist ja auch ein Pfeil. (Und der Trick bei Penrose ist ja, dass er dann irgendwie die Skala ändert.)

    Vergiss das Konzept der virtuellen teilchenpaare einfach, das ist besser 😉 (Erklärung siehe die letzten Teile der QFT-Serie.)
    Auf der Ebene der Quantenprozesse sind alle Prozesse (außer dem Messprozess) reversibel und können genausogut andersrum ablaufen.

  64. #65 Niels
    23. September 2012

    @Alderamin
    Danke fürs Rauskopieren und Zusammenfassen.

    unsere Gehirne erwüchsen vollkommen ausgeformt aus dem Chaos

    Das ist ja das bekannte “boltzmann brain paradox”..
    Ich sehe nicht, wie die Inflation dieses Problem lösen würde?

    Übrigens gibts Aufzeichnungen, wie es wirklich begann 😉 :
    https://abstrusegoose.com/strips/blip.PNG

  65. #66 MartinB
    23. September 2012

    @Niels
    Gilt das Boltzmann-Brain Paradox eigentlich als Gegenargument gegen das anthropische Prinzip für die1e500 Welten der Stringtheorie? (Weil ja ein Universum, in dem sich bloß unser Sonnensystem per Fluktuation gebildet hat, von der Entropie her wesentlich wahrscheinlich ist als das das wir sehen?)

  66. #67 MartinB
    23. September 2012

    @Alderamin
    Laut Penrose (Emperors New Mind) löst die Inflation das Problem nicht, weil ein hochentropische Zustand durch bloßes Aufblasen nicht plötzlich niederentropisch wird und weil es dann immer noch wesentlich wahrscheinlicher wäre, dass die niedrige Entropie nur in einem winzigen Bereich des Universums zu sehen wäre.

  67. #68 Alderamin
    23. September 2012

    @Niels, MartinB

    Greene schreibt dazu im Kapitel “Entropie und Inflation” auf S. 358, dass die Inflation zum einen die Entropie erhöht hat, indem sie ca. 10^80 Teilchen im beobachtbaren Universum erzeugt hat, jedoch dadurch, dass diese Teilchen fast völlig gleichförmig verteilt wurden, für eine sehr geringe Gravitationsentropie gesorgt hat. Netto kommt zwar eine Entropiezunahme heraus, aber diese ist viel geringer, als wenn die entstandene Materie von vorneherein verklumpt gewesen wäre. Dadurch, dass die Teilchen aufeinander zu fallen können, können sie Arbeit leisten, z.B. Sterne bilden und darin durch Kompressionswärme die Fusion zünden. Ich zitiere nochmal eine Stelle (S. 359, 2. Absatz):

    Dadurch, dass der Inflationsausbruch den Raum glättete und für ein homogenes, gleichförmiges, niederentropes Gravitationsfeld sorgte, erzeugte er allerdings, und das ist der entscheidende Punkt, eine gewaltige Lücke zwischen dem Beitrag zur Entropie, den die Gravitation tatsächlich beisteuerte, und dem, den sie hätte leisten können. Zwar wuchs während der Inflation die Gesamtentropie an, jedoch um einen armseligen Betrag, verglichen mit der Zunahme, die tatsächlich hätte erzielt werden können. Insofern hat die Inflation ein niederentropisches Universum geschaffen: am Ende der Inflation hatte die Entropie zwar zugenommen, aber nicht im Entferntesten um den Faktor, um den die räumliche Ausdehnung angewachsen war.

    Also wurde der Raum insgesamt eigentlich gar nicht niederentropischer als vorher. Damit begründet sich auch, warum die gewaltige Hitze beim Urknall entstehen musste, um die Abnahme der Gravitationsentropie zu überkompensieren (aus dieser Energie kondensierte dann die Materie auf einer Energiestufe, die noch weitere Fusion bis hin zum Eisen erlaubt).

    Um die Inflation anzustoßen, brauchte es wiederum nur eine statistische Schwankung, die in einem winzigen Raumbereich die Dichte so weit erhöhte, dass dort ein falsches Vakuum entstand, in dem das Inflatonfeld einen höher angeregten Zustand hatte. Wenn man in einem hochentropischen Raum eine sehr, sehr lange Zeit wartet, wird sich so ein Zustand irgendwann einmal lokal zufällig einstellen. Auf jeden Fall mit viel höherer Wahrscheinlichkeit, als ein komplett konfiguriertes Universum mit Beoboachtern und deren komplett gefälschten Erinnerungen.

    So verstehe ich Brian Greene. Ihr solltet das Buch wirklich mal lesen, eine so zusammenhängende Beschreibung der Welt habe ich vorher noch nirgends gelesen. Der Mann ist einer der führenden String-Theoretiker und nicht irgendein Amateur.

  68. #69 Niels
    23. September 2012

    @MartinB

    Gilt das Boltzmann-Brain Paradox eigentlich als Gegenargument gegen das anthropische Prinzip für die1e500 Welten der Stringtheorie?

    Sollte man meinen und einige Kosmologen sehen das auch so.

    Die Stringtheoretiker behaupten allerdings frech das Gegenteil.
    Siehe z.B. hier:
    https://arxiv.org/abs/0812.0005v1
    We study the recently proposed “stationary measure” in the context of the string landscape scenario. We show that it suffers neither from the “Boltzmann brain” problem nor from the “youngness” paradox
    Es gibt auch noch andere Argumente, warum dieses Paradoxon angeblich durch die String-Landschaft vermieden wird.
    Ich versteh kein einziges davon.

    @Alderamin

    Wenn man in einem hochentropischen Raum eine sehr, sehr lange Zeit wartet, wird sich so ein Zustand irgendwann einmal lokal zufällig einstellen. Auf jeden Fall mit viel höherer Wahrscheinlichkeit, als ein komplett konfiguriertes Universum mit Beoboachtern und deren komplett gefälschten Erinnerungen.

    Das “boltzmann brain paradox” ist doch genau das Gegenargument dazu, dass unser Universum nur eine statistische Schwankung darstellt.
    Kleine Entropieschwankungen sind sehr viel wahrscheinlicher als große Entropieschwankungen.
    Eine Schwankung, aus der heraus unser Universum entstehen könnte, ist also viel unwahrscheinlich als eine solche, aus der ein Boltzmann-Gehirn entsteht.
    Es geht eben gerade nicht um ein
    komplett konfiguriertes Universum mit Beoboachtern und deren komplett gefälschten Erinnerungen,
    sondern im Extremfall um ein “Universum”, das aus einem einzigen Gehirn besteht.
    Ich hab unter meinen Lesezeichen diesen Artikel von Sean Carroll gefunden, in dem es exakt um diesen Punkt geht.
    https://blogs.discovermagazine.com/cosmicvariance/2008/01/14/boltzmanns-universe/
    Dort wird das genauer ausgeführt, deswegen brauch ich das nicht tun. 😉

    Von Carroll gibt es auch noch einen Artikel, in dem das Konzept des Boltzmann-Gehirns ziemlich gut erklärt wird:
    https://blogs.discovermagazine.com/cosmicvariance/2006/08/01/boltzmanns-anthropic-brain/

    Damit die Inflation passend stattfindet, braucht man doch sogar noch niederentropischere Anfangsbedingungen als im “althergebrachten” Urknall-Modell.
    Inflation kann das Problem also meiner Meinung nach nicht lösen, sondern verschärft es sogar noch zusätzlich.

  69. #70 Alderamin
    23. September 2012

    @Niels

    Was ist jetzt daran unwahrscheinlicher, dass in einer winzig kleinen Raumregion die von Guth verlangte Dichte entsteht, um die Inflation auszulösen, als ein komplett funktionierendes Gehirn eines Freak-Observers? Dichteschwankungen gibt’s im Vakuum doch sowieso. Die Schwankung braucht “nur” groß genug zu sein, um die Inflation auszulösen. Wenn man sehr, sehr, sehr lange wartet (oder sehr, sehr, sehr viel Raum hat), dann tritt eine Schwankung beliebig hoher Größe irgendwann (irgendwo) mit Sicherheit auf.

    Eine Gaußkurve hat einen Definitionsbereich von -unendlich bis plus unendlich. Für jeden Wert des Definitionsbereichs gibt es eine Wahrscheinlichkeit > 0 und einen endlichen Erwartungswert für dessen Eintreten.

  70. #71 Alderamin
    23. September 2012

    @Niels

    In dem ersten von Dir zitierten Paper scheint die zu widerlegende Annahme die zu sein, dass das Universum ohne Inflation einfach so aus einer statistischen Schwankung des Vakuums entstanden zu sein. Das könnte in der Tat unwahrscheinlich sein, aber es lässt m.E. außer Acht, dass die Inflation nur einen submikroskopischen Bereich zum Start benötigt, der sich dann sehr rasch vergrößert. Dabei entsteht irgendeine zufällige Anordnung zufälliger Teilchen, die an und für sich eine sehr hohe Entropie hat (und damit nicht extrem unwahrscheinlich ist). Lediglich ist sie sehr gleichförmig verteilt, was eine niedrige Gravitationsentropie bedeutet. Daraus können dann Mannigfaltigkeiten von Galaxienanordnungen entstehen.

  71. #72 Niels
    24. September 2012

    @Alderamin

    Das könnte in der Tat unwahrscheinlich sein, aber es lässt m.E. außer Acht, dass die Inflation nur einen submikroskopischen Bereich zum Start benötigt, der sich dann sehr rasch vergrößert.

    Der “normale” Urknall benötigt doch ebenfalls nur einen submikroskopischen Startbereich.
    Wo liegt der Unterschied, außer das der Inlfations-“Zustand” eine noch geringere Entropie hat und damit noch unwahrscheinlicher ist?

    Dichteschwankungen gibt’s im Vakuum doch sowieso. Die Schwankung braucht “nur” groß genug zu sein, um die Inflation auszulösen. Wenn man sehr, sehr, sehr lange wartet (oder sehr, sehr, sehr viel Raum hat), dann tritt eine Schwankung beliebig hoher Größe irgendwann (irgendwo) mit Sicherheit auf.

    Eine Gaußkurve hat einen Definitionsbereich von -unendlich bis plus unendlich. Für jeden Wert des Definitionsbereichs gibt es eine Wahrscheinlichkeit > 0 und einen endlichen Erwartungswert für dessen Eintreten.

    Schon klar. Das ist der Ausgangspunkt.

    Die Frage ist doch, ob ein Boltzmann-Gehirn oder ein passendes Inflations-Universum wahrscheinlicher ist.
    Über die Problematik wird eigentlich erst seit weniger als zehn Jahren geforscht, verschiedene Forscher haben hier unterschiedliche Meinungen.
    Hängt natürlich damit zusammen, dass niemand weiß, wie man die Entropie des Universums richtig berechnet.
    Bis jetzt gibt es dazu noch richtige keine Mehrheitsmeinung.
    Es beschäftigen sich aber auch nicht besonders viele Leute damit, das ist sogar für theoretische Physiker ein bisschen zu abgefahren.

    Noch mal Carroll dazu:

    The inflationary paradigm has been very successful in many ways. Its predictions of slight deviations from perfect uniformity agree with observations of density variations in the universe. As an explanation for time asymmetry, however, cosmologists increasingly consider it a bit of a cheat, for reasons that Roger Penrose and others have emphasized. For the process to work as desired, the ultradense dark energy had to begin in a very specific configuration. In fact, its entropy had to be fantastically smaller than the entropy of the hot, dense gas into which it decayed. That implies inflation has not really solved anything: it “explains” a state of unusually low entropy (a hot, dense, uniform gas) by invoking a prior state of even lower entropy (a smooth patch of space dominated by ultradense dark energy). It simply pushes the puzzle back a step: Why did inflation ever happen?

    One of the reasons many cosmologists invoke inflation as an explanation of time asymmetry is that the initial configuration of dark energy does not seem all that unlikely. At the time of inflation, our observable universe was less than a centimeter across. Intuitively, such a tiny region does not have many microstates, so it is not so improbable for the universe to stumble by accident into the microstate corresponding to inflation.

    Unfortunately, this intuition is misleading. The early universe, even if it is only a centimeter across, has exactly the same number of microstates as the entire observable universe does today. According the rules of quantum mechanics, the total number of microstates in a system never changes. (Entropy increases not because the number of microstates does but because the system naturally winds up in the most generic possible macrostate.) In fact, the early universe is the same physical system as the late universe. One evolves into the other, after all.

    Among all the different ways the microstates of the universe can arrange themselves, only an incredibly tiny fraction correspond to a smooth configuration of ultradense dark energy packed into a tiny volume. The conditions necessary for inflation to begin are extremely specialized and therefore describe a very low entropy configuration. If you were to choose configurations of the universe randomly, you would be highly unlikely to hit on the right conditions to start inflation. Inflation does not, by itself, explain why the early universe has a low entropy; it simply assumes it from the start.

    Mir ist jetzt aber nicht wirklich klar, warum die Quantenphysik aussagt, dass die Anzahl der Mikrozustände des Universums konstant sein muss…

    Das Argument von Penrose, warum Boltzman-brains (BB) wahrscheinlicher sind, geht wieder ein bisschen anders.
    Ich finde das Statement im Moment aber leider nicht mehr. Mal schaun, ob ich das durch ein bisschen Nachdenken noch zusammenbringe.

    Ein zusätzliches Problem sind übrigens die BBs in unserem eigenen Universum. Wenn unsere heutiges kosmologisches Modell stimmt, dehnt sich das Universum ewig aus, die Entropie steigt und irgendwann ist kein “normales” Leben (also keine durch die Evolution entstandenen sogenannten ordinary observers = OOs) mehr möglich.
    Da das Universum unendlich weiterexistiert, kommen dann irgendwann unendlich viele solcher BBs auf jeden einzelnen OO, der jemals existiert hat.

  72. #73 MartinB
    24. September 2012

    @Niels
    Danke für die Erläuterungen, das passt alles halbwegs zu meinen Vorstellungen.

  73. #74 Alderamin
    24. September 2012

    @Niels

    Berücksichtigt Carroll denn auch, dass durch die entstandenen Teilchen die Entropie in Summe ansteigt?

    Außerdem: für die Inflation gibt es ja auch gewisse Indizien, die Penrose erst mal erklären muss. Die Feinstruktur der kosmischen Hintergrundstrahlung soll doch Quantenfluktuationen aus der Zeit vor der Inflation widerspiegeln. Und mit der Dunklen Energie sehen wir aktuell einen ähnlichen, nur viel gemächlicheren Prozess ablaufen. Wenn aber die Inflation stattgefunden hat, dann hat sie die Gravitationsentropie gesenkt, indem sie das Weltall glatt gezogen hat.

    Was ist eigentlich aus den Ideen geworden, dass Schwarze Löcher neue Universen hervorbringen könnten? Die würden doch auch die Materiedichte lokal auf extrem hohe Werte heben, was nach Guth eigentlich reichen müsste, um ein falsches Vakuum und damit eine kosmische Inflation zu erzeugen. Ist das mittlerweile vom Tisch?

  74. #75 Erik Ihle
    24. September 2012

    @MartinB
    “… ist ja, dass man, damit Zeit “vergehen” kann, ein weiteres Bezugssystem bräuchte, damit man von “außen” draufgucken kann.”
    “Laut Penrose (Emperors New Mind) löst die Inflation das Problem nicht, weil ein hochentropische Zustand durch bloßes Aufblasen nicht plötzlich niederentropisch wird …”

    Ein “neues” Bezugssystem, welches Entropie verringern kann, wäre ein Lösungsansatz. Ich habe es mal auf meiner Timeline auf facebook skizziert.

    @Alderamin
    Auch die “gewaltige” Lücke des Beitrages zur Entropie, welche die Gravitation beisteuern könnte, kann geschlossen werden.

    Verzeiht mir meine amateurhafte Art meiner Darstellungen.
    Viele liebe Grüsse . . . ..

  75. #76 Niels
    24. September 2012

    @Alderamin

    Berücksichtigt Carroll denn auch, dass durch die entstandenen Teilchen die Entropie in Summe ansteigt?

    Klar. Er hat übrigens vor kurzem ein Buch veröffentlicht From Eternity to Here: The Quest for the Ultimate Theory of Time, in dem er seine Ideen dazu ausführlich darstellt.
    Ich hab es aber nicht gelesen.

    für die Inflation gibt es ja auch gewisse Indizien, die Penrose erst mal erklären muss.

    Praktisch alle an dieser Diskussion Beteiligten glauben, dass es eine Inflationsphase gegeben hat.
    Es geht vielmehr darum, auf welche Weise unser Universum entstanden ist bzw. wie der Zustand vor der Entstehung unseres Universums aussah. Das BB-Paradoxon wurde erfunden, um zu argumentieren, dass bestimmte Urzustände unplausibel wären.
    Zusätzlich hofft man, dass man durch die Klärung dieser Frage einen Ursache für die Existenz des Zeitpfeils findet.

    Carrolls Lösung ist zum Beispiel
    that inflation occurs asymptotically both forwards and backwards in time, implying a universe that is (statistically) time-symmetric on ultra-large scales.
    Aber frag mich nicht nach Einzelheiten.

    Was ist eigentlich aus den Ideen geworden, dass Schwarze Löcher neue Universen hervorbringen könnten?

    Das schwirrt immer noch durch den Raum, ist aber gerade nicht die neuste Mode und damit uninteressant.
    Bei der Universumentstehung hat ja mittlerweile beinahe jeder der auf diesem Gebiet Forschenden seine eigene Privat-Hypothese.

    Ich kenn mich da aber nicht wirklich gut aus, verlass dich da nicht zu sehr auf mich.
    Das ist mir alles eigentlich noch viel zu schwammig und irgendwie auch zu wenig wissenschaftlich, um da wirklich tief einzusteigen.
    Das sind ganz nette und anregende Ideen, aber wenn man ehrlich ist, muss man mindestens auf eine funktionierende Quantengravitation warten, damit das mehr als bloßes Herumraten werden kann.
    Es gibt noch so viel “harte” Wissenschaft, die ich noch nicht verstehe. Im Allgemeinen finde ich es lohnender, mich mit sowas zu beschäftigen. Da hab ich dann eher das Gefühl, dass ich wirklich etwas über das Universum gelernt habe.

    Deswegen besitze ich nur sehr oberflächliches Wissen zu den hier besprochenen Themen. Das reicht mir.
    Aber dadurch ist es eben auch sehr gut möglich, dass ich dabei Unsinn erzähle.

  76. #77 Alderamin
    24. September 2012

    Deswegen besitze ich nur sehr oberflächliches Wissen zu den hier besprochenen Themen.

    Oberflächlich, öhem, hüstel…

    Was machst Du eigentlich vom Job her, wenn ich fragen darf? Postdoc oder Prof an einer Uni? Schwerpunkt Astrophysik?

  77. #78 AndreasM
    24. September 2012

    @Niels: Eine Frage zu den Mikrozuständen.
    Wenn man etwas über Mikrozustände im Gesamtsystem Universum aussagen will, dann sind das wohl alle möglichen Zustände, die das Universum annehmen kann und das bildet dann eine Art Phasenraum. Die Naturgesetze bestimmen die Übergangswahrscheinlichkeiten in diesem Phasenraum.
    Kann man sicher sein, dass sich hier nicht aus wahrscheinlicheren Trajektorien Strukturen bilden, die manche Mikrozustände wahrscheinlicher als andere machen?
    Könnte also ein BB, das kein OO ist, unwahrscheinlicher sein als ein OO, obwohl es in mehr möglichen Mikrozuständen vorkommt, diese aber nicht in einer Art Wahrscheinlichkeitsfokus im Phasenraum liegen?

  78. #79 Niels
    24. September 2012

    @Alderamin
    Kosmologie und ART interessieren mich nur als Hobby, da besitze ich wirklich keine besonderen Qualifikationen.
    Und mit dem Zeitpfeil und den BBs hab ich mich wirklich nur sehr oberflächlich beschäftigt. Das ist keine falsche Bescheidenheit oder gar ein Fishing for Compliments.
    Ich finde es eben ziemlich wichtig, dass man klarstellt, ob man etwas wirklich verstanden hat oder ob man mehr oder weniger Hörensagen verbreitet.

    Beruflich beschäftige ich mich als Experimentalphysiker mit Anregungen von Bose-Einstein-Kondensaten.
    Mit der Doktorarbeit bin erst vor ziemlich kurzer Zeit fertig geworden und bin gerade im ersten Drittel meiner ersten befristeten Zwei-Jahres-Postdoc-Stelle.

    Momentan schieb ich es noch vor mir her, aber demnächst muss ich mich wirklich mal ernsthaft mit meiner Lebensplanung auseinandersetzen. Leider siehts nämlich so aus, als ob da auch in mindestens mittelfristiger Zukunft immer wieder nur befristete Zwei-Jahres-Stellen drin sind.
    Ein Kollege hat sogar gerade einen 6-Monats-Vertrag bekommen und soll sich in dieser Zeit selbst um Drittmittel bemühen. Großes Kino.
    Ist schon echt toll, wie man in Deutschland mit den Nachwuchswissenschaftlern umgeht!

    @AndreasM

    Kann man sicher sein, dass sich hier nicht aus wahrscheinlicheren Trajektorien Strukturen bilden, die manche Mikrozustände wahrscheinlicher als andere machen?

    Warum sollten denn bestimmte Mikrozustände nicht wahrscheinlicher sein als andere? Es liegt doch kein abgeschlossenen Systems im Gleichgewicht vor, oder?
    Ich hab aber echte Probleme, mir den Phasenraum eines expandierendes Universum mit dunkler Energie (und zusätzlicher Vakuumenergie(?)) vorzustellen.
    Ich verstehe ehrlich gesagt nicht einmal, warum Carroll sich so sicher ist, dass die Anzahl der Mikrozustände eines solchen Universums konstant sein muss.

    Könnte also ein BB, das kein OO ist, unwahrscheinlicher sein als ein OO, obwohl es in mehr möglichen Mikrozuständen vorkommt

    Ob das zutrifft ist ja gerade eine der zu beantwortenden Fragen.
    (Übrigens kann ein BB niemals ein OO sein und auch umgekehrt nicht, wenn ich die Begriffe richtig verstehe.)

  79. #80 Niels
    24. September 2012

    Übrigens ist sich Carroll sicher, dass wir keine BBs sind.
    Das würde nämlich nicht mit unseren Beobachtungen übereinstimmen.

    Ich weiß, da begeben wir uns jetzt in die Niederungen der Philosophie, aber meiner Meinung nach ist unmöglich, durch Beobachtungen festzustellen, ob man ein BB ist oder nicht.
    Mache ich da einen Denkfehler?

  80. #81 MartinB
    24. September 2012

    @Niels
    “wirklich keine besonderen Qualifikationen.”
    Hüstel, vielleicht sind deine Maßstäbe ein bisschen andere als die der meisten hier 😉

    “Ist schon echt toll, wie man in Deutschland mit den Nachwuchswissenschaftlern umgeht!”
    Jawohl, das ist es. Meiner Ansicht nach eine unglückliche Kombination aus opferbereiten Jung-Wissenschaftlern, finanziell eher verarmten oder am Wohl ihrer Mitarbeiter wenig interessierten Institutsleitern und generellen strukturellen Problemen (zu wenig tenure tracks etc.)

    “meiner Meinung nach ist unmöglich, durch Beobachtungen festzustellen, ob man ein BB ist oder nicht.
    Mache ich da einen Denkfehler?”

    Naja, wäre nicht auch bei einem Boltzmann Brain ein “kleineres” Universum wahrscheinlicher? Allein die ganzen Beobachtungsdaten der Astronomen sind ja für reines Bewusstsein eher überflüssig, ebenso wie unglaublich viele andere Dinge (600000 Käferarten)? Schlägt da das Wahrscheinlichkeitsargument nicht genauso zu? Oder verstehe ich das Boltzmann-Brain-Argument falsch?

  81. #82 Alderamin
    24. September 2012

    @Niels

    Ich finde es eben ziemlich wichtig, dass man klarstellt, ob man etwas wirklich verstanden hat oder ob man mehr oder weniger Hörensagen verbreitet.

    Dann ist bei mir alles Hörensagen 😉 Ich hab’ zwar die Grundvorlesungen Physik gehört und sogar drei Semester Astronomie (als Nebenfach), bin aber ansonsten “nur” promovierter Informatiker.

    Übrigens ist sich Carroll sicher, dass wir keine BBs sind.
    Das würde nämlich nicht mit unseren Beobachtungen übereinstimmen.

    Das ist aber sehr inkonsequent. Wenn man einerseits argumentiert, die seien viel wahrscheinlicher als ein ganzes Weltall, dann müsste man konsequenterweise auch annehmen, man sei ein BB. Und wie er es unterscheiden will? Eigentlich dürfte er es nicht können. Man könnte argumentieren, wenn man etwas lernt, das man noch nicht wusste, das man dann durch Herleitung erst als richtig erkennt (z.B ein mathematischer Satz), dann spricht das zumindest dafür, dass es draußen noch eine Welt gibt, wo andere auch nachdenken. Natürlich könnte der ganze Prozess auch falsch erinnert sein. Ich hab’ mal in einer Spektrum gelesen, man sei in der Philosophie heute über Descartes hinaus, ich erinnere mich aber nicht mehr genau, inwiefern.

    @MartinB

    Allein die ganzen Beobachtungsdaten der Astronomen sind ja für reines Bewusstsein eher überflüssig, ebenso wie unglaublich viele andere Dinge (600000 Käferarten)?

    Man braucht ja nur einen Beobachter durch das BB zu erklären, und genau dessen Wissenstand. Da niemand 600000 Käferarten kennt, braucht derjenige nur eine handvoll Käfer zu kennen und sich einzubilden, es gäbe noch 599995 andere.

    Trotzdem erscheint mir ein funktionierendes Gehirn mit Erinnerungen an Käfer, andere Leute, das Internet, und allerlei Zeugs das wunderbar logisch zusammen passt und das sich kein einzelner Mensch (wobei ein BB kein Mensch wäre) hätte ausdenken können, einfach so spontan entstanden, irgendwie unwahrscheinlicher als ein winzig kleines Volumen falsches Vakuum mit nichts anderem als einer sehr hohen Energiedichte, ohne jegliche Struktur, aber vermutlich verstehe ich da auch was falsch.

  82. #83 Niels
    25. September 2012

    Na ja, es ging ja um berufliche Qualifikationen.
    An der Uni war ich zwar in einer ART-Vorlesung, allerdings kann ich da nicht mal einen Schein vorweisen. Da hab ich wenig verstanden, die Übungsblätter nicht bearbeitet und auch die Klausur nicht mitgeschrieben.
    In der Kosmologie-Vorlesung war ich sogar nur die ersten paar Wochen. Die hatte nämlich offiziell die ART-Vorlesung als Voraussetzung und das war leider auch ernst gemeint.

    Ich finde es schwierig, auch noch bei den verschiedenen BBs mit Wahrscheinlichkeiten zu hantieren. Ich hab keine Ahnung, wie man da vernünftige Zahlen abschätzen könnte. Ich kenne nämlich zum Beispiel auch nur ein Dutzend Käfer.
    Ist aber mit Sicherheit ein möglicher Ausweg, danke für den Hinweis.
    Wobei es aber auch Rechnungen wie in diesem Paper gibt, nach denen die Wahrscheinlichkeit für eine Inflations-Universum-Fluktuation so unfassbar viel kleiner als die Wahrscheinlichkeit für eine Standard-Urknall-Universum-Fluktuation ist, dass das alle Abwägungen über die spezielle Besonderheit “meines” BB totschlagen müsste.
    (Wobei dort ein Argument mit
    At this point we will add one more assumption, which is motivated by the success of string theory
    eingeleitet wird. Dann muss man sich wahrscheinlich doch keine Sorgen machen. 😉 )

    Darüber hinaus gibt es noch ein weiteres Problem:
    Wenn ich wirklich ein BB bin, kann ich eigentlich überhaupt keine Wahrscheinlichkeiten vergleichen. Schließlich kenne ich dann ja gar keine echten Fakten bzw. Beobachtungen. Wie das echte Universum aussieht, dass mich hervorgebracht hat, kann ich niemals wissen.
    Die Kosmologie “meiner Erinnerungen” hat wahrscheinlich überhaupt nichts mit der “echten” Kosmologie zu tun, deswegen ist jede Rechnung und jeder Vergleich eigentlich sinnlos.

    Der Tipp mit Descartes ist aber sehr gut, Alderamin.
    Das ist eigentlich ziemlich genau das “Böse Geist”-Problem Descartes bzw. im modernen philosophischen Sprachgebrauch das sogenannteGehirn im Tank Gedankenexperiment.
    Hätte mir eigentlich auffallen müssen. Das hab ich mir vor ner ganzen Weile mal angeschaut. Die Argumente der Philosophen, warum ich kein Tank-Gehirn sein kann, haben mich damals überhaupt nicht überzeugt. (Und wenn da jemand mit Dualismus argumentiert, lese ich gar nicht erst weiter. 😉
    Mit dem Stichwort “Gehirn im Tank” habe ich übrigens sofort einen Spektrum-Artikel dazu gefunden, der sogar kostenlos runterladbar ist:
    https://www.spektrum.de/alias/serie-philosophie-teil-6-skepsis-und-wissen/was-koennen-wir-von-der-welt-wissen/1067446
    War der gemeint, Alderamin? Ich finde ihn wenig nutzbringend.

    Hat Dennett dazu eigentlich auch etwas geschrieben, MartinB?
    Das ist eigentlich der Philosoph, mit dem ich noch am meisten anfangen kann. Wobei ich noch nicht besonders viel von ihm gelesen habe.

    Wobei wir jetzt aber sehr, sehr weit im Offtopic angekommen sind…

  83. #84 Erik Ihle
    25. September 2012

    @MartinB
    “”meiner Meinung nach ist unmöglich, durch Beobachtungen festzustellen, ob man ein BB ist oder nicht.
    Mache ich da einen Denkfehler?””

    Selbsterkenntnis vermag feststellen, ob jemand ein Boltzmann-Brain ist.
    – – Subjektivität muss mit Objektivität übereinstimmen: DIFF=0
    – – Entropie =0 im Blockuniversum des BB
    – Informationen im Universum kann ein BB nicht “ausgrenzen”

    Mit meiner Rotstiftskizze vom 24.09.2012 will ich dieser Fragestellung einmal nachgehen. Ich glaube mit dieser Fragestellung zeigen zu können, das die Eulersche Identität als Weltformel bezeichnet werden kann.

    . . . .. wünschen Sie mir Glück und Zeit – Herr Bäker – . . . ..

  84. #85 AndreasM
    25. September 2012

    @Niels: Wenn nicht alle Mikrozustände im Universum (über unendlich Zeit gesehen) gleich wahrscheinlich sind, dann steigt die Entropie des Universums auch nicht immer an.
    Und man weiss damit auch nicht, ob ein BB wahrscheinlicher als ein OO ist.

    Wenn man ein BB wäre, dann wären ja alle Erinnerungen eingebildet und wie soll man da überhaupt irgendetwas feststellen können. Ich könnte in diesem Moment entstanden sein, nur in diesem Moment denken und im nächsten schon wieder vergehen.

  85. #86 MartinB
    25. September 2012

    @Niels
    Kann ich mich im Moment nicht entsinnen, dass Dennett etwas dazu geschrieben hätte – wahrscheinlich ist ihm das egal. (Wenn ich mich recht entinne, gibt es ein oder zwei Hirn-im-Tank-Geschichten von ihm im Buch “The Mind’s I”, das er mit Hofstadter geschrieben hat, ist aber ewig her, das ich das gelesen habe, und da ging es auch um andere Fragen (was ist Bewusstsein)).

  86. #87 Alderamin
    25. September 2012

    @Niels

    Die Philosophieserie in SdW war’s, aber nicht dieser Artikel. Vielleicht habe ich die Schlussfolgerung, man sei weiter als Descartes, auch nur falsch erinnert, in diesem Artikel hier steht ja eher das Gegenteil. Mein Argument gegen Descartes (allerdings nicht 100%ig gegen ein BB, dass ja quasi nur einen kurzen Moment zu existieren braucht) habe ich oben genannt: man kann etwas lernen, das man noch nicht wusste und (im Falle von mathematischen Beweisen in jedem Fall) im Nachhinein verifizieren. Demnach gibt es “draußen” jemand, der Dinge weiß, die ich vorher nicht wusste.

    Aber Philosoph bin ich noch weniger als Kosmologe, als Vorsicht, Hörensagen! 😉

  87. #88 Niels
    26. September 2012

    @Alderamin

    man kann etwas lernen, das man noch nicht wusste und (im Falle von mathematischen Beweisen in jedem Fall) im Nachhinein verifizieren

    Hm, muss ich mal drüber nachdenken. Das Problem ist wahrscheinlich, dass man immer argumentieren kann, dass man sich daran nur erinnert.

    @MartinB

    wahrscheinlich ist ihm das egal

    Wahrscheinlich ist das die richtige Einstellung.

    @AndreasM

    Wenn nicht alle Mikrozustände im Universum (über unendlich Zeit gesehen) gleich wahrscheinlich sind, dann steigt die Entropie des Universums auch nicht immer an.

    Versteh ich nicht ganz.
    Woraus soll das folgen? Irgendwie aus der Ergodenhypothese oder aus dem Liouville-Satz?
    Kannst du das noch ein bisschen ausformulieren oder anders ausdrücken? In der statistischen Mechanik bin ich nicht mehr wahnsinnig sattelfest.

    Die Entropie des Universums steigt doch außerdem “über unendlich Zeit gesehen” nicht immer weiter an. Oder mindestens ist es unklar, ob das so ist.
    Wenn das Universum im Wärmetod / Big Freeze endet, erreicht das Universum irgendwann maximale Entropie.
    https://en.wikipedia.org/wiki/Heat_death_of_the_universe
    (Das ist natürlich wieder umstritten. Es gibt auch Forscher, bei denen es gar keine maximale Entropie gibt.
    Bei Penrose ist es sogar noch seltsamer, nach seiner Idee wird die Entropie dann sogar wieder “zurückgesetzt” und es gibt einen neuen Urknall. Dazu hat Martin erst vor kurzem etwas geschrieben:
    Roger Penroses zyklisches Universum)

  88. #89 Frink
    Berlin
    26. September 2012

    @MartinB

    Schon rein mathematisch sieht man, dass da etwas faul ist – eine Sekunde pro Sekunde lässt sich als Bruch kürzen zu 1. Damit hat das Vergehen der Zeit eine Geschwindigkeit, die keine Einheit hat, was schon mal merkwürdig ist.

    Ja, das stimmt, aber dann darf man doch weder bei der Landkarte noch bei dem Buch die Einheit kürzen oder?

    Wenn ihr das Buch in 20 Stunden gelesen habt, dann verging die Zeit in dem Buch für euch mit einer mittleren Geschwindigkeit von 6 Monaten pro 20 Stunden.

    Müsste man nicht eher von
    6 Monate (System 1) / 20 Stunden (System 2)
    sprechen?
    Sonst kommt als “Einheit” ja wieder 1 heraus.

    Auf jeden Fall ein anregender Artikel,
    Danke & Grüße,
    Frink

  89. #90 MartinB
    26. September 2012

    @Frink
    Das ist jetzt wohl Geschmackssache – bei der Landkarte darf ich das rauskürzen, weil letztlich die Karte ja in demselben Raum existiert wie das dargestellte Objekt. Bei der Lesegeschwindigkeit wäre deine Schreibweise sicher korrekter, aber es ging ja nur um die Illustration (implizit habe ich das ja gemacht, indem ich eben nicht gekürzt habe).

  90. #91 KeinAnfang
    27. September 2012

    auch die Idee, dass wir ein Bewusstsein haben, dass einem Körper innewohnt, ist meiner Ansicht nach eine Fiktion

    Es wäre nett, wenn du darüber schreibts. Ein bisschen extraphilosopische Philosophie ist ja nicht schlecht 😉
    Denn ich würde sagen, dass NUR in einem Körper ein Bewusstsein Innewohnt. Da ja das Bewusstsein eine Immanente Eigenschaft des Körpers ist.

  91. #92 Marel
    28. September 2012

    Martin Bäker schrieb:

    “Damit der “Jetzt-Pfeil” eine Geschwindigkeit haben kann, bräuchten wir eine zweite, andere Zeit, in der wir die Geschwindigkeit messen könnten. Die gibt es in unserer Welt allerdings nicht. Ein Wesen, das außerhalb unserer Welt steht, könnte eine eigene, unabhängige Zeit haben, und in der würde die Aussage Sinn ergeben.”

    Und:

    “Noch schwieriger wird die Sache, wenn wir Quanteneffekte ins Spiel bringen. Nehmen wir zwei mit einander quantenverschränkte Teilchen, die jemand in der Mitte zwischen Alpha Centauri und der Erde erzeugt und dann zu mir und meinem Brieffreund schickt. Solange die Eigenschaften der Quantenteilchen nicht gemessen wurden, sind sie in einem Überlagerungszustand (…). Wenn einer von uns beiden die Eigenschaften seines Teilchens misst, dann sind damit die Eigenschaften des anderen Teilchens festgelegt. Ich könnte also mein Teilchen beispielsweise heute um 11:00Uhr messen, mein Brieffreund dann um 12:00Uhr (unter Verwendung unserer koordinierten Uhren). Die Logik würde uns sagen, dass es meine Messung war, die den Verschränkungszustand zerstört hat, denn sie fand ja zuerst statt. Für den Beobachter in der Andromeda-Galaxis dagegen sieht die Sache andersherum aus – er nimmt wahr, dass es die Messung bei Alpha Centauri war, die zuerst stattfand und deshalb für die Aufhebung der Verschränkung verantwortlich war. (…)

    Das sind wirklich interessante Gedanken und sie werden in der “Eisenstädter Deutung der Quantenmechanik” (ED) auf die Spitze getrieben.

    Die Argumentation beginnt mit Zeilinger (“Einsteins Schleier”), wo er Interferenz am Gitter für C60 – Fullerene nachweist und damit zeigt, dass Quantenobjekte auch sehr groß sein können, wie dies Schrödinger bereits vermutet hat (Schrödingers Katze, die im Zustand der Superposition zwischen lebend und tot verharrt, bis jemand nachschaut, ob das radioaktive Teilchen zerfallen ist und Gift freigesetzt hat oder nicht. Übrigens ist das unnötig grausam. In meiner Version wird Futter freigesetzt und die Katze verharrt im Zustand hungrig/satt). Zeilinger meint, ein Mensch mit Lebenserhaltungssystem könne prinzipiell ein Schrödingerkatzensystem bilden, was dieser Mensch dann aber erlebt, sei fraglich.

    Dies ist der Angelpunkt der ED. Die nimmt ein Relativitätsprinzip an: salopp gesagt, die Physik muss überall gleich sein und auch Experimente müssen überall die gleichen Ergebnisse liefern, egal ob im Katzensystem oder außerhalb. Der Außenstehende stellt fest, dass für ihn die Gesetze der klassischen Physik gelten, das Katzensystem aber im Zustand der Superposition verharrt, bis es zur Beobachtung kommt. Also stellt der Mensch im Katzensystem fest, dass für ihn die Gesetze der klassischen Physik gelten, während die Außenwelt im Zustand der Überlagerung aller möglichen Ereignisse verharrt.

    Wie ist das möglich? Nach der ED ist die Quantenmechanik die Theorie des Trennens und Verschmelzens von Kosmen und beschäftigt sich nur deshalb mit kleinen Sachen, weil die leichter zu isolieren sind. Das Katzensystem ist ein Kosmos für sich, ohne Kontakt zu sonst was. Das gleiche gilt für die Außenwelt. In beiden läuft die Zeit in klassischer Weise ab, aber es gibt keinen Informationsaustausch, keine Möglichkeit die Uhren aufeinander zu eichen. Das ist der Grund, warum innerhalb seines Kosmos jeder den Eindruck hat, bei ihm laufe die Zeit ganz normal, aber beim anderen verharre sie und alles was passieren kann passiere dann “auf einmal”, im Augenblick des Kontakts zwischen den Kosmen, wenn die Uhren wieder verglichen werden können. Man bedenke dabei, dass es keine Referenzuhr gibt, keine absolute Zeit, die sich außerhalb der Kosmen befindet, denn außerhalb gibt es nichts.

    Die ED ist wahrscheinlich auch nicht sinnvoller als die Kopenhagener Deutung, aber sie führt vor Augen, wie wenig wir über die Zeit eigentlich wissen. Natürlich hätte sie ganz komische Konsequenzen, wenn sie richtig wäre (z. B: damit die Wellenfunktion nicht kollabiert, muss es nicht wirklich überhaupt keinen Informationsfluss zwischen den Kosmen geben, sondern eben nur so wenig, dass der Fluss der Zeit nicht mehr aufeinander abgestimmt werden kann usw.)…

    So Sabine, da hast Du Deine Katze.

  92. #93 MartinB
    28. September 2012

    Ist das schon wieder eine privat-Theorie? Oder ist die “Eisenstädter Deutung” irgendwo publiziert?

  93. #94 Alderamin
    28. September 2012

    @Niels

    Das Problem ist wahrscheinlich, dass man immer argumentieren kann, dass man sich daran nur erinnert.

    Deswegen meinte ich, dass dies kein 100%iges Argument gegen ein BB sei, das nur einen Moment existiert und nur diesen Moment lang den Eindruck hat, irgendetwas zu wissen. Kennt man ja auch vom Déjà Vue: man meint sich zu erinnern, irgendetwas genau so schon einmal erlebt oder geträumt zu haben, “erinnert” sich in Wahrheit jedoch nur an das, was man gerade eben gesehen hat.

    Beim “normalen” Descartes bzw. dem Gehirn im Tank wird, so wie ich das verstehe, vorausgesetzt, dass man eine Weile existiert und sich in Ruhe Gedanken machen kann. Dann und nur dann greift das Lernargument.

  94. #95 Alderamin
    30. September 2012

    @Niels

    Ich hab’ den Spektrum-Artikel gefunden (hab’ auch noch das Heft; war aber unterwegs). Gibt es auch zum kostenlosen Download (anscheinend die ganze Serie, sollte ich mir mal rein-, äh, runterziehen). Hat nicht direkt mit BBs und Gehirnen im Tank zu tun, ist aber ganz interessant zu lesen, und wirft Descartes vor, den Dualismus als gegeben zu betrachten, was man heute nicht mehr annimmt; das war wohl der Punkt, an den ich mich erinnerte.

    Hier ist der Artikel.

  95. #96 Niels
    30. September 2012

    @Alderamin
    Danke für den neuen Spektrum-Artikel.
    Ich gehe prinzipiell davon aus, dass die dualistische Position falsch ist. Und das Thema der Ich-Wahrnehmung spielt in unserem Fall eigentlich keine Rolle, oder?

    Beim “normalen” Descartes bzw. dem Gehirn im Tank wird, so wie ich das verstehe, vorausgesetzt, dass man eine Weile existiert und sich in Ruhe Gedanken machen kann. Dann und nur dann greift das Lernargument.

    wenn man etwas lernt, das man noch nicht wusste, das man dann durch Herleitung erst als richtig erkennt (z.B ein mathematischer Satz), dann spricht das zumindest dafür, dass es draußen noch eine Welt gibt, wo andere auch nachdenken.

    Ja, aber hier wird die “äußere Welt” doch schon vorausgesetzt?
    Sie umfasst eben mindestens den “Bösen Geist” bzw. den Tank samt zugehöriger Rechenmaschine.
    Diese Akteure versorgen das Gehirn ja aktiv mit Sinneswahrnehmungen.
    Dann kann auch man noch einen Schritt weitergehen und fragen, woher sie ihre Informationen bekommen bzw. bekommen haben usw.
    Oder hab dieses Gedankenexperiment falsch verstanden?

    @MartinB
    Hast du verstanden, was AndreasM mit seinen über unendliche Zeit betrachteten Mikrozuständen sagen wollte?

  96. #97 MartinB
    30. September 2012

    @Niels
    Nein, habe ich nicht.

  97. #98 Alderamin
    30. September 2012

    @Niels

    Und das Thema der Ich-Wahrnehmung spielt in unserem Fall eigentlich keine Rolle, oder?

    Nö, höchstens am Rande, wir sind da ein wenig abgedriftet, eigentlich ging’s mir ja nur darum, dass Desacartes überholt sei, laut Spektrum-Artikel, der jetzt gefunden ist, aber darüber hinaus nicht wirklich was zum Thema beiträgt.

    Ja, aber hier wird die “äußere Welt” doch schon vorausgesetzt?
    Sie umfasst eben mindestens den “Bösen Geist” bzw. den Tank samt zugehöriger Rechenmaschine.
    Diese Akteure versorgen das Gehirn ja aktiv mit Sinneswahrnehmungen.
    Dann kann auch man noch einen Schritt weitergehen und fragen, woher sie ihre Informationen bekommen bzw. bekommen haben usw.
    Oder hab dieses Gedankenexperiment falsch verstanden?

    Beim “Gehirn im Tank” geht’s wohl mehr darum, ob unsere Wahrnehmung echt ist oder vorgetäuscht. Natürlich gibt’s in diesem Gedankenexperiment auch eine Außenwelt, die aber mit der scheinbaren, wahrgenommenen Welt nichts zu tun haben muss. Wir könnten dann nicht erkennen, ob wir real wären oder nicht.

    Bei Descartes und seinem Dualismus geht man nicht notwendigerweise von einer Außenwelt aus, sondern der Wahrnehmende könnte eine körperlose Seele sein, aber auch ein träumender Riese oder ein Gehirn im Tank oder eine Maschine. Hier geht’s mehr darum, worüber man sich sicher sein kann und worüber nicht. Descartes sagt, man könne sich nur seiner selbst sicher sein. Mit dem Lern-Argument stimmt das nicht mehr, allerdings kann man sich nicht sicher sein, ob derjenige, von dem man lernt, auch so aussieht, wie man ihn wahrnimmt, oder ob er nur eine Simulation ist. Jedenfalls kann man feststellen, dass man nicht alleine ist und es irgendeine Außenwelt geben muss.

    Beim Boltzmann-Brain kommt verschärfend hinzu, dass es spontan aus irgendetwas entsteht und ebenso schnell wieder vergeht, es existiert möglicherweise nur einen kurzen Moment. Jegliche von ihm wahrgenommene Vergangenheit ist irreal und hat nie statt gefunden, der Input, den es gelernt zu haben glaubt, ist ebenfalls falsch. Wenn man ein BB ist, weiß man nichts über seine Umwelt; alles was man zu wissen glaubt, ist ein Zufalls-Erinnerungskonstrukt. Deswegen kann ein BB auch nicht berechnen, wie wahrscheinlich sein Zustandekommen wäre, selbst wenn es die Zeit dazu hätte. Wenn ich ein BB wäre, dann gäbe es allerhöchstwahrscheinlich keinen Rechner vor mir und keinen Niels, keine Scienceblogs. Es gäbe keine Erde, keine Sonne und keine Sterne, es gäbe kein Periodensystem der Elemente, keine Atome, keine Nukleonen. Es gäbe irgend etwas mir völlig unbekanntes, das zufällig ein Gehirn mit Bewusstsein entstehen lassen könnte, aber vermutlich von komplett anderer Natur als das wäre, was ich meine gelernt zu haben. Insofern könnte ich niemals abschätzen, ob meine Existenz wahrscheinlicher oder weniger wahrscheinlich als die Entstehung eines Universums wäre, weil ich nicht wüsste, was ein Universum in Wirklichkeit wäre und woraus es besteht.

    Deswegen kann man sich niemals sicher sein, dass man kein BB ist, und man kann nicht einmal die Wahrscheinlichkeit dafür abschätzen. Man kann bestenfalls argumentieren, dass bei einer zufälligen Entstehung eines Gedächtnisinhalts in den seltensten Fällen ein konsistenter Inhalt herauskommen sollte, sondern eher recht wirres Zeug, so dass man als BB eher erwarten könnte, keine konsistente Erinnerung zu haben und eine solche folglich eher einem OO zuzuschreiben wäre. Auf der anderen Seite gibt es auch viel mehr Gehirne von Tieren als Menschen und viel mehr Gehirne von Menschen, die ganz andere Probleme als Philiosophie und Kosmologie haben, und mit dem gleichen Argument müsste man eigentlich ein Insekt sein oder ein Homo Erectus oder wenigstens ein chinesischer Bauer. Wie man sieht, ist das Argumentieren mit Wahrscheinlichkeiten nicht so ganz aufschlussreich (das ist jetzt eine Steilvorlage für das anthropische Prinzip).

    Daher mag mich auch das Argument nicht so recht überzeugen, ein BB sei wahrscheinlicher als ein Urknall. Ein BB weiß das ohnehin nicht – und ein OO sollte dann eigentlich ein BB sein oder irgend etwas häufigeres als ein gebildetes Mitglied der westlichen Zivilisation. Oder mit seiner Wahrscheinlichkeitsabschätzung einfach falsch liegen.

  98. #99 Marel
    1. Oktober 2012

    M. Bäker schrieb:

    “Ist das schon wieder eine privat-Theorie? Oder ist die “Eisenstädter Deutung” irgendwo publiziert?”

    Spielt das irgendeine Rolle?
    Ich lese genau wie Ihr alle regelmäßig “Nature”, “Science” & “Spektrum der Wissenschaften” und da steht im Wesentlichen schon alles drin, was in diesem Forum diskutiert wird (die seltenen Ausnahmen sind’s, auf die man hofft). Interessanter sind für mich neue Gedanken.

    Ein Beispiel dafür, was mich eigentlich an Deiner Frage stört aus einem anderen Bereich: In Deinem Beitrag: “Lückenfüller der Evolution ” beziehst Du Dich hauptsächlich auf einen Artikel über Strudiella devonica in Nature 448, 2 August 2012, S. 82 ff. Da ich den gelesen habe und auch die Supp. Info. dazu (und auch die anderen Arbeiten, auf die Du Dich inhaltlich beziehst) bietet mir Dein Beitrag nichts neues. Ich hätte trotzdem dazu schreiben können, z. B., dass man die Insektenfundlücke für real hält, weil das erste Insekt, dass man danach findet, Delitzschala bitterfeldensis, nicht nur bereits drei Paar wunderschön entwickelte Flügel hat sondern einen hochgradig abgeleiteten Saugapparat usw. (davor hat es keine geflügelten Insekten gegben). Das hätte ich auch tun dürfen, denn es ist eine allgemein bekannte Tatsache. Aber gerade deshalb langweilig. Ich hätte lieber geschrieben, dass mir die sogenannten Fühler der Strudiella verdächtig nach Carapaxrand aussehen und dass ich daher nicht glaube, dass es sich wirklich um ein Insekt handelt. Das aber ist “schon wieder eine Privattheorie” und hat damit in Deinem Forum nichts verloren?

    Als Du Zeilingers “Einsteins Schleier” gelesen hast und an die Stelle geraten bist, wo er erwähnt, dass man im Prinzip einen Menschen mit Erhaltungssystem in eine “Schrödingerkatze” verwandeln könne, aber man natürlich nicht weiß, was dieser Mensch dann erlebt, hast Du nicht angefangen darüber nachzudenken, was er wahrnimmt? Natürlich hast Du, niemand ist so verkrustet, dass er (oder sie) das nicht tun würde! Aber weil Du Deine Gedanken dazu nicht veröffentlicht hast, darfst Du darüber hier nicht schreiben? Sind das Deine Spielregeln?

    Verständnis habe ich hingegen dafür, wenn Du (statt “schon wieder Privattheorie”) schreibst, dass Dich diese Frage überhaupt nicht interessiert. Das ist jedem seine Privatsache, da kann man nichts machen, o.k., dann eben keine Diskussion.

  99. #100 MartinB
    1. Oktober 2012

    @Marel
    Das mit der “Privattheorie” bezieht sich darauf, dass ich, seit ich diesen Blog schreibe, etwa alle 4 Wochen einen Kommentar oder eine mail von irgendwem bekomme, der mal wieder die Weltformel gefunden, die relativitätstheorie widerlegt oder etwas ähnliches getan hat. Ich habe keine Zeit und Lust, mir so etwas anzugucken. Gegen begründete Spekulationen habe ich gar ichts und hätte die auch bei dem Insektenartikel gern gelesen.

    Ich habe auch hier nichts gegen Spekulationen, aber wenn ich von einer “Eisenstädter Deutung” lese, in der “Kosmen verschmelzen” (was immer das bedeuten soll) und die anscheinend nicht publiziert ist (ansonsten bitte Referenzen liefern, dann kann man die Dinge nachlesen), die von dir aber als eine gültige Interpretation präsentiert wird, dann werde ich eben misstrauisch.

    Zeilingers “Einsteins Schleier” habe ich gar nicht gelesen, insofern habe ich mir die Frage beim lesen auch nicht gestellt.

  100. #101 Marel
    2. Oktober 2012

    @MartinB

    Niemand schreibt absichtlich Schwachsinn, aber Schwachsinn ist auch relativ. Um wieder auf die Insektenstammesgeschichte zurück zu kommen: wer gerade einmal einen Artikel über Strudiella gelesen hat wird bezüglich dieses Bereichs sicherlich etwas anderes als Schwachsinn empfinden als jemand, der auch noch, sagen wir mal, “History of the Insects”, “Evolution of the Insects” und den Insektenkästner gelesen hat (sehr dicke Wälzer). Aber! Und das ist das Schöne an der Entomopaleontologie, die russischen Wissenschaftler (“History of …”) halten vieles von dem, was die angloamerikanischen (“Evolution of …”) schreiben auch für Schwachsinn und umgekehrt! Auch große Sachkenntnis ist also kein Garant, dass man weiß was Schwachsinn ist und was nicht.
    Eine Deutung ist keine Hypothese, keine Weltformel, sie ist einfach nur Unfug und auch die Kopenhagener Deutung ist meines Wissens nie in einem Peer Review Magazin erschienen (“The ‘Kopenhagener Deutung’ of Quantum Mechanics”, by Niels Bohr …)
    “Kosmos” war lediglich ein kürzeres Wort für “Schrödinger Katzen System beliebiger Größe” (man kann es auch SKS nennen) und die Überlegung war einfach die, dass der Mensch mit Lebenserhaltungssystem im SKS (Alice) eigentlich in der gleichen Situation ist, wie der Beobachter (Bob) draußen, denn der eine ist von der Welt des anderen genauso isoliert wie der andere von der des einen. Daher sollte er (also sie) auch “das Gleiche” erleben, also sich selbst in einer Welt der klassischen Physik, während Alice, wenn es wieder zu Kontakt zur Außenwelt kommt (das ist mit verschmelzen gemeint) erleben sollte, dass es Bob war, der in Superposition verharrte, während Bob natürlich in diesem Augenblick erlebt, dass es Alice war, bei der, solange der Kontakt nicht vorhanden war, im Zustand der Überlagerung (z. B. zwischen hungrig und satt, wenn Alice eine Katze ist und der Whiskasdosenöffner mit einem radioaktiven Teilchen gekoppelt ist, das zerfallen kann oder nicht) verblieben ist.
    Ist es nicht erstaunlich, dass so etwas einfaches wie völlige Isolation eines Teils der Welt dazu führt, dass er plötzlich quantenmechanische Eigenschaften aufweist? Ist es nicht nahe liegend zu fragen, wieso das sein kann? Und ist es dann nicht auch – wenn schon nicht nahe liegend dann möglich – zu vermuten, dass das eben daran liegen könnte, dass der eine keine Information über den Zeitverlauf beim anderen mehr erhalten kann, weil sie ihre Uhren nicht mehr abgleichen können und deshalb einen anderen Eindruck über den Zeitverlauf dort erhalten?
    Es ist natürlich durchaus möglich, dass solche Überlegungen in die Rubrik “Perpetuum mobile” und “Widerlegung der Relativitätstheorie” fallen und nichts mehr zu tun haben mit “begründeter Spekulation”, aber dann wüsste ich gerne, wieso? Sicherlich sind sie aber genauso unnütz, das muss ich schon zugeben.

  101. #102 MartinB
    2. Oktober 2012

    @Marel
    Erstens: Wenn ich hier Blödsinn schreibe (gerade in Gebieten, in denen ich kein echter Experte bin), dann nehme ich Korrekturen immer gern entgegen.

    Zweitens: Natürlich kann man so spekulieren, wie du es tust, und ich habe auch gar nichts dagegen. Wenn du hier aber aufschlägst mit dem Begriff “Eisenstädter Deutung” und damit den Eindruck erweckst, als sei das eine genau so gut durchdachte und anerkannte Deutung der Qm wie die Kopenhagener oder jede andere, dann hat das eine andere Qualität als wenn du geschrieben hättest: “Ich habe mir da was überlegt, was haltet ihr davon”. (Und wen du dann noch das Wort “Kosmos” in eher idiosynkratischer Weise benutzt, macht das die Sache nicht besser.)

    Wie gesagt, ich bekomme (gerade in letzter Zeit) dauernd irgendwelche merkwürdigen Privattheorien zugeschickt.

    So, jetzt aber zum Kern – Deine Idee ist also, dass man eine Deutung braucht, die immer symmetrisch funktioniert, bei der also immer der “Beobachter” sich selbst als klassisches System wahrnimmt und seine Um gebung als Überlagerung, habe ich das richtig verstanden? Mir ist jetzt gar nicht klar, was das mit dem “Zeitablauf” zu tun hat oder haben soll, denn das QM-Messproblem gibt es ja auch in einer nicht-relativistischen QM. Darüber hinaus ist mir nicht klar, ob eine solche Deutung konsistent möglich ist – wie geht denn das mit zwei verschränkten Photonen in einem EPR-Experiment? Was ist denn für die Photonen der Überlagerungszustand der Außenwelt? Unterschiedliche Stellungen der Polarisatoren? Aber warums ollten die in einer qm-Überlagerung existieren?

  102. #103 Niels
    2. Oktober 2012

    @Alderamin
    Wie Descartes Argument genau lautet weiß ich nicht. Da hast du sicher recht.

    Im Übrigen sind wir völlig einer Meinung, kann ich alles unterschreiben.

  103. #104 MartinB
    2. Oktober 2012

    Das Descartes-Argument geht in etwa so (meiner Erinnerung an die Meditationen nach, ist aber über 20 Jahre he, deckt sich aber mit dem, was bei Wikipedia steht https://de.wikipedia.org/wiki/Meditationes_de_prima_philosophia.):
    ich denke, also bin ich. (Bis dahin können die meisten noch folgen, allerdings nicht jeder (Kant z.B., soweit ich ihn verstehe).
    Ich bin, also komme ich irgendwo her.
    Ich komme irgendwo her, also wurde ich geschaffen.
    ich wurde geschaffen, also gibt es einen Gott.
    Als mensch bin ich unvollkommen, also kann ich mir keinen vollkommenen Gott ausdenken, wenn es keinen gibt.
    Ich kann mir einen vollkommenen Gott ausdenken, also gibt es ihn auch.
    Weil Gott vollkommen ist, wird er mich nicht täuschen.
    Da Gott mich nicht täuschen wird, ist die ganze Welt real.

    Nun ja, so viel zu Descartes…

  104. #105 Marel
    3. Oktober 2012

    @ MartinB

    Niemand hat Dir vorgeworfen, dass Du Blödsinn geschrieben hast, sondern mir (und das wahrscheinlich auch zu Recht, ich will’s bloß begründet haben). Im Gegenteil bin ich der Meinung, dass Deine Beiträge didaktisch hervorragend sind und auch inhaltlich passen. Aber Du bist klüger – auf einem Gebiet wo Du nicht sattelfest bist, äußerst Du keine eigene Meinung.

    Ja, wenn Bob den Eindruck hat, Alice sitze in einem SKS, muss auch Alice den Eindruck haben, Bob, der in der Außenwelt ist, sitze in einem SKS. Der Grund dafür ist, dass die Gesetze der Physik überall gleich sein müssen und die Ursache des Phänomens – Isolation von allen Wechselwirkungen – für beide die Gleiche ist. Also wenn wir z. B. Deinen Beitrag über Wellenfunktion, Teil 8, nehmen; das Elektron, dessen Wellenfunktion an einer Barriere gespalten wird. Dann müsste sich aus Sicht des Elektrons die Wellenfunktion der Restwelt aufspalten (ich glaube nicht, dass diese Doppelsicht konsistent ist, weil in die Wellenfunktion nur Größen des beobachteten und nicht auch des beobachtenden Systems eingehen, dennoch sollte es eigentlich so sein).

    Das Kappen aller Wechselwirkungen mag praktisch fürchterlich schwierig zu erreichen sein, theoretisch ist es aber ganz simpel. Wie kann so etwas simples einen derartigen Effekt haben? Mit Relativität hat das jedenfalls nichts zu tun. Gekappt wird zwischen den zwei Systemen jedenfalls auch die Möglichkeit, den Zeitverlauf zu vergleichen. Stell Dir einen Soldaten vor, dem beim Exerzieren Augen und Ohren verbunden sind, der wird ziemlich schnell aus dem Gleichschritt kommen. Nur dass er in diesem Fall stehen bleibt, allerdings nicht an einem Ort, sondern an einem Zeitpunkt (nämlich dem Zeitpunkt, wo wieder Kontakt zwischen Alice und Bob hergestellt wird und ein Zeitvergleich wieder möglich wird).

    Wie Du weißt, wie alle wissen, gibt es einen fundamentalen Unterschied zwischen Raum und Zeit. Nehmen wir an, Quasimodo in seinem Raumschiff treibt mit gleichmäßiger Geschwindigkeit durchs All (mit einer Raumachse). Raum vergeht, Zeit vergeht, super. Jetzt will er an einem Ort verharren, was zwar einigen Aufwand macht aber möglich ist. Dann läuft die Zeit weiter und wenn er wirklich in perfekter Starre verharrt, spielt es keine Rolle, ob die Zeitpunkte wie auf einer Perlenschnur aufeinander folgen oder irgendwie über einen Bereich verschmiert sind. Dann fliegt Quasi weiter und beschließt, abermals stehen zu bleiben, nur diesmal an einem Zeitpunkt. Aber er muss feststellen, dass das nicht geht.
    In der Welt der klassischen Physik. Wir können uns trotzdem vorstellen, was dann passiert, indem wir die beiden Achsen vertauschen. Wenn dann der Ort weiterläuft so wie üblicherweise die Zeit, wird der Ort am Zeitpunkt verschmieren.

    Dieses Gleichnis von Quasimodo erklärt natürlich nicht, warum zum Zeitpunkt des Kontaktes, wenn für den Außenstehenden alles auf einmal passiert, dies in Form einer Überlagerung aller Möglichkeiten passieren muss.

    Ok. ich gebe zu, dass Du mir jetzt mit Recht vorwerfen kannst, dass ich Schwachsinn schreibe. Aber wenn ich einfach so akzeptieren soll, dass ein C60-Molekül (oder was Größeres) auf magisch-mystische Weise beim Durchgang durch ein Gitter in Superposition mit sich selbst gerät, nur weil keiner hinschaut, dann dreht sich mir der Magen um.

  105. #106 MartinB
    3. Oktober 2012

    @Marel
    “(ich glaube nicht, dass diese Doppelsicht konsistent ist”
    Ich auch nicht, insofern verstehe ich den Ansatz nicht so ganz.

    ” Jetzt will er an einem Ort verharren, was zwar einigen Aufwand macht aber möglich ist.”
    Aber nur, wenn er irgendwie einen Bezug zu einem anderen Objekt herstellt. Es gibt nicht einen festen Ort im All, den gibt esnicht mal in der newtonschen Physik, wegen der Galilei-Invarianz. Raumpunkte haben keine feste Identität, insofern macht es keinen Unterschied, ob Quasimodo mit konstanter Geschwindigkeit fliegt oder auf der Stelle steht.

    Was das Stehenbleiben an einem Zeitpunkt angeht – im Blockzeit-Bild ist das überhaupt kein konzeptionelles Problem. Es gibt eine Weltlinie, die durch die Raumzeit verläuft, das ist es. Raum und Zeit unterschieden sich nur darin, dass Weltlinien immer Raumartig sein müssen (oder lichtartig, wenn du ein Photon bist).
    Es ergibt für mich keinen Sinn, an einem Zeitunkt “stehenzubleiben” – denn der Begriff impliziert ja, dass für dich die Zeit trotzdem irgendwie weiterlaufen soll, nur die äußere zeit nicht. Die beste Annäherung daran wäre, sich an einem Zeitpunkt einfach in Luft aufzulösen, dann endet deine Weltlinie und ist zeitlich begrenzt (so wie sie räumlich begrenzt ist, wen du irgendwo stehenbleibst (im passenden Bezugssystem)).

    Das Argument: “Zwei Systeme, die voneinander isoliert sind, können ihren Zeitablauf nicht synchronisieren, deswegen muss die Isolation was mit der Zeit zu tun haben” überzeugt mich nicht besodners – zwei Systeme, die isoliert sind, können ja alles mögliche nicht abstimmen, nicht nur den Zeitablauf (sondern auch z.B. keinen Raum-Maßstab oder eine Massenskala oder sonst irgendwas…)

    “Aber wenn ich einfach so akzeptieren soll, dass ein C60-Molekül (oder was Größeres) auf magisch-mystische Weise beim Durchgang durch ein Gitter in Superposition mit sich selbst gerät, nur weil keiner hinschaut, dann dreht sich mir der Magen um.”
    Akzeptieren solltest du das, weil es nun mal nach allem, was wir wissen, so ist – aber nicht unbedingt “einfach so”. Ich hoffe ja auch noch darauf, dass man eines Tages eine erweiterte Quantenmechanik findet, wo solche Dinge besser verstanden werden. Lies mal das aktuelle Heft von Spektrum der Wissenschaft, das hat einen guten Artikel dazu.

  106. #107 Frank Wappler
    ...and.a.reference.for.the.rest.of.us
    3. Oktober 2012

    MartinB schrieb (#106, 3. Oktober 2012):
    > im Blockzeit-Bild […] Raum und Zeit unterschieden sich nur darin, dass Weltlinien immer Raumartig sein müssen

    &#1F631; !

    p.s.

    Frank Wappler schrieb (#25, 20. September 2012):
    > […] Zugrundelegung der entsprechenden Messdefinitionen der RT und Betrachtung von geeigneten Näherungen (insbesondere [tex] \beta \ll 1 [/tex] bzw. [tex] lim \beta \rightarrow 0 [/tex])

    … Betrachtung von geeigneten Näherungen (insbesondere
    \beta \ll 1 bzw. lim \beta \rightarrow 0 ).

  107. #108 Marel
    3. Oktober 2012

    @MartinB
    “ich glaube nicht, dass diese Doppelsicht konsistent ist”
    Ich auch nicht, insofern verstehe ich den Ansatz nicht so ganz.”

    Solange die Wellenfunktion bloß hingeschrieben ist und nicht hergeleitet, besteht immer die Hoffnung, daß sie bloß ein Spezialfall ist, z.B. für unendlich große Systeme des Beobachtenden. Dann könnte diese Sicht schon konsistent sein – vielleicht.

    Über Deine Einwände, den Zeitpunkt betreffend, bin ich Dir dankbar, das ist richtig – was ich gesagt habe, stimmt nicht.

    Also benimmt sich das C60 Molekül wie ein Kind, daß so lange es beobachtet wird brav ist und schlimm, sobald kein Erwachsener herschaut …. wirklich schwer zu akzeptieren.
    Danke auch für den Hinweis den SdW-Artikel betreffend, ich hab’ seit etwa 2 Jahren SdW nicht mehr selbst abonniert und muß es mir ausborgen aber sobald ich es bekomme (in einem Monat) werde ich den Artikel lesen. Bin schon neugierig!

    Vielen Dank für Deine Zeit, jetzt weiß ich warum das Unfug ist, was ich geschrieben habe.

  108. #109 Marel
    4. Oktober 2012

    @MartinB

    Eine kleine Ergänzung muss ich doch noch anbringen: es stimmt zwar, dass für Quasi das am Ort verharren (weil nicht unterscheidbar von konstanter Geschwindigkeit) oder am Zeitpunkt verharren keinen Sinn macht, weil er kein Bezugssystem hat, aber für das eigentliche Problem ist das nicht wichtig, denn da gibt es natürlich einen zeitlichen Bezug: der Augenblick, in dem die Wechselwirkung zwischen Bob und Alice abbricht. Und sogar einen zweiten, nämlich der, wo (eigentlich wann) sie wieder beginnt. Was den Raummaßstab betrifft, magst Du auch recht haben, die Frage ist nur ob das nachher (wenn wieder Wechselwirkung da ist) irgendwelche Folgen hat (ich weiß es nicht).
    Mit der Masse ist das was anderes, ich denke da an das Delayed-Choice-Experiment in extremster Form: Ein Quasar sendet ein Photon aus, das durch eine Gravitationslinse so abgelenkt wird, dass es aus zwei Richtungen (nach Milliarden Jahren) zu uns kommt und dann hier zur Interferenz gebracht wird. Wenn man an Gravitonen glaubt, dann wechselwirkt das Photon die ganze Zeit mit ihnen, sonst würde es nicht von der Gravitationslinse abgelenkt werden und geht doch nicht in ein Klassische-Physik-Verhalten über. Ich versteh nichts von Gravitonen, aber ich denke mit Gravitonen wechselwirken hat was mit Massemessen zu tun. Wenn das so ist, dann kann ein System auch dann SKS bleiben, wenn Information über seine Masse an die Außenwelt gelangt (schließlich geht ja die Masse des beobachteten Systems auch in die Wellenfunktion ein und das setzt doch gleiche Skala voraus, oder nicht?).
    Deutungen sind nicht nur Spaß; eine gute Deutung sollte es erlauben, die Wellenfunktion oder etwas Allgemeineres, das die Wellenfunktion als wichtigen Spezialfall enthält, herzuleiten (richtig herzuleiten, nicht nur hinzuschreiben). Keine Deutung, ob Kopenhagen (“Kein Phänomen ist ein Phänomen, außer es ist ein beobachtetes Phänomen” – Niels Bohr), Everett, Dekohärenz, schafft das bisher und sie sind alle ziemlich … hmmmm merkwürdig, aber wenigstens konsistent. Vielleicht klärt sich das Ganze für mich ein bisschen wenn ich den SdW-Artikel bekomme (am 11.11).

  109. #110 MartinB
    4. Oktober 2012

    @Marel
    Nein, eine Deutung der Qm kann nicht erlauben, etwas herzuleiten, was im Formalismus der QM nicht drin ist – dann ist es keine Deutung mehr, sondern eine umfassendere Theorie.

    Ich fürchte, den Rest deines Posts habe ich nicht so ganz verstanden – weder das mit dem zeitlichen Bezug, noch das mit dem Quasar.

  110. #111 Fritzchen
    Frankfurt
    4. Oktober 2012

    Warum es nicht eine Metapher, sondern eine grundlose Erfindung ist zu sagen die Zeit vergeht. Zeit bewegt sich nicht. So vermutete auch mal der Hobbyphilosoph EvG.
    Vllt gefällt es ja dem einen oder anderen.

  111. #112 Aveneer
    4. Oktober 2012

    Ich habe für mich ja einen anderen Weg gefunden. 🙂

    Man nehme eine Ebene und zeichne ein paar Teilchen rein. So wie bei dir Martin eine Jetzt-Eben nur mit Teilchen.

    Dann nimmt man einen Zeitpfeil und stellt jeweils einen senkrecht über jedes Teilchen. Sind ein paar mehr wie bei dir Martin, aber das macht ja nichts. Jedes Teilchen hat ja auch seinen Eigenen?

    Anstatt nun die Ebene mit den Teichen diesem Zeitpfeil „Blockweise“ in die Zukunft zu schicken (das Bild von dir Martin), kippe ich die Zeitpfeile um 90°. Nun liegen alle Zeitpfeile auf dieser Ebene der Teilchen. Und man kann „sehen“, wie jedes Teilchen seinem eigenen Zeitpfeil auf der Ebene folgt. Diese Ebene nenne ich die Jetzt-Ebene.

    Gruß, Aveneer
    PS: Und alle bewegen sich demanch “nach Higgs” alle mit c in Richtung Zeit ;-o Manche nur “hin und her”

  112. #113 MartinB
    5. Oktober 2012

    @Aveneer
    Entweder ich habe das nicht verstanden, oder du hast gerade die Konstruktion einer gleichzeitigskeitsebene in der SRT hingeschrieben…?

  113. #114 Aveneer
    5. Oktober 2012

    Hallo Martin,

    Das was ich hingeschrieben habe, das ist die eine Ebene in der sich alle Teilchen befinden – das was wechselwirkt.

    Auch bei dir liegen alle Teilchen zu einem Zeitpunkt “x” auf einer dieser Ebenen-Oder? Da wo du dein „Jetzt“ hingeschrieben hast – diese eine Ebene. Was enthält deine “Jetzt-Ebene” außer den Ereignissen? Nicht alle Teilchen des Universums? Jetzt mal unabhängig ihrer Bewegung im Raum. Existieren tun sie doch gleichzeitig. Ich glaube kaum, dass ein Teilchen in der Vergangenheit und ein Anderes in der Zukunft existiert?

    Wenn zu einem Zeitpunkt „x“ (dein „Jetzt“ im Bild) kein Teilchen unseres gesamten Universums in der Vergangenheit oder Zukunft existiert (auf einer der anderen Ebenen deiner Zeichnung) – wo sind sie dann?

    Und wenn die Teilchen diese gemeinsame Ebene nicht verlassen können, dann müssen sie sich für mich auf dieser einen Ebene bewegen.

    Ich finde es zumindest sehr anschaulich? Die Teilchen bewegen sich entlang des Zeitpfeils auf dieser “Jetzt-Ebene”.

    Zur SRT…

    Da der Zeitpfeil bei dieser Betrachtung “deckungsgleich” zur Bewegungsrichtung ist (Teilchen folgt der Zeit) also zu v ist – und wir nach Higgs uns alle mit c bewegen (also die elementarsten Teilchen natürlich) – bewegen sie sich mit c in Richtung Zukunft (und damit wir ja irgendwie auch). Damit sind die Pfeile „v“ und „t“ deckungsgleich und gleich lang.

    Was mich tatsächlich an…
    Epstein,
    „Es gibt nur eine Geschwindigkeit. Alles, wir mit eingeschlossen, bewegt sich stets mit Lichtgeschwindigkeit.“
    Denken lässt.

    Während wir auf dem Stuhl ruhen, bewegen sich unsere Teilchen mit c durch diese Ebene. Und wenn wir aufstehen, dann bewegen wir uns durch den Raum und die Teilchen aus denen wir bestehen. Zusammen immer mit c.

    Das ist total SRT konform (mathematisch) -Glaub ich.

    Also ich finde das Bild – toll?
    Kein zuckeln keine Blöcke – ein stetiges Jetzt (von außen betrachtet)

  114. #115 MartinB
    5. Oktober 2012

    @Aveneer
    Ja, mein Bild ist gültig für eine newtonsche Sicht der Dinge (weil einfacher).
    Dein Bild verstehe ich imemr noch nicht – wenn die Teilchen sich “in der Ebene” bewegen, wo ist dann die zusätzliche Dimension für die Zeit?

    Das mit dem “Alles bewegt sich mit c” ist in meinen Augen ein bisschen Problematisch – es funktioniert, wenn man im Teilchenbild denkt, aber nicht mehr, wenn man alle Teilchenbilder (Feynmandiagramme) überlagert – in einem (massiven, wegen der Bindungsenergie) Proton habe ich z.B. das Gluonenfeld. Ich kann das zwar als überlagerung aus unendlich vielen Einzel-Gluonen-Prozessen denken (wo sich in jedem einzelnen Bild alle Gluonen mit c bewegen), aber in der Summe kann das Feld trotzdem ruhen. (Und auf der kleinen Längenskala haben die Gluonen außerdem noch eine endliche Wahrscheinlichkeit, sich nicht mit c zu bewegen.) Deswegen ist die Aussage zwar plakativ und eingängig, aber mit seeehr viel Vorsicht zu genießen.

  115. #116 Aveneer
    5. Oktober 2012

    @Martin
    [ZITAT] Ja, mein Bild ist gültig für eine newtonsche Sicht der Dinge (weil einfacher).[/ZITAT]
    Aber ein ruckeln und zuckeln durch den Lauf der Zeit ist doch für keinen wirklich vorstellbar und entspricht nicht dessen was wir messen und wahrnehmen. Damit konnte ich noch nie leben – einfacher?
    [ZITAT] Dein Bild verstehe ich imemr noch nicht – wenn die Teilchen sich “in der Ebene” bewegen, wo ist dann die zusätzliche Dimension für die Zeit? [/ZITAT]
    Puh – jetzt wo du es sagst? In deinem Kopf? 😉
    Habe die Zeit „durch bewegen mit c im „Jetzt“ ersetzt (kippen der Zeit um 90°). Komme dann mit etwas Hilfe von Epstein im Bereich der SRT gut zurecht.
    [ZITAT] Das mit dem “Alles bewegt sich mit c…..(wo sich in jedem einzelnen Bild alle Gluonen mit c bewegen), aber in der Summe kann das Feld trotzdem ruhen. [/ZITAT]
    Du hast schon recht. Es kommt aber weniger darauf an, ob dass Modell (sofort) mit allen Modellen (insbesondere quantenmechanisch, wo Zeit eine andere Rolle spielt?) 100% zusammen passt.

    Das Proton ist eine „Einheit“ in dem sich alles mit c durch den Raum bewegt und das Proton bewegt sich zusätzlich selbst durch den Raum: Zusammen mit c. Ob das für alle Modelle der richtige Weg ist?

    Für den Alltag reicht es und dass obwohl ich damit leben „muss“(kann/darf), dass Zeit nicht in dem Sinne existiert – wie Masse – nur eine Illusion ist. Und trotzdem bewegen sich Uhren relativ bei mir……
    [ZITAT] Deswegen ist die Aussage zwar plakativ und eingängig, aber mit seeehr viel Vorsicht zu genießen. [/ZITAT]
    Mit seeeehr viel Vorischt?
    Was soll ich sage?
    Zuuu ssspppäätt :-)))))

  116. #117 Aveneer
    5. Oktober 2012

    Ich weis [ZITAT][/ZITAT] funktioniet nicht – aber was soll man sonst machen? Geht doch

  117. #118 MartinB
    5. Oktober 2012

    @Aveneer
    Was “ruckelt und zuckelt” denn da?

    “Habe die Zeit „durch bewegen mit c im „Jetzt“ ersetzt (kippen der Zeit um 90°). Komme dann mit etwas Hilfe von Epstein im Bereich der SRT gut zurecht.”
    Vermutlich habe ich heute nen Brett vor dem Kopf – ich raff’s nicht.

    “Das Proton ist eine „Einheit“ in dem sich alles mit c durch den Raum bewegt”
    Eben nicht, das versuche ich ja gerade zu sagen. Einzelne reelle Elementarteilchen bewegen sich mit c; schon in Feynmandiagrammen muss man aber alle Amplituden mitnehmen, auch die, die sich mit mehr als c oder weniger als c bewegen, und wenn man die Quantenfelder statt der teilchen betrachtet (was das korrektere Bild liefert), dann bewegt sich nicht mehr alles mit c.

  118. #119 Aveneer
    5. Oktober 2012

    Hmm:
    Es ist doch eigentlich völlig egal wie schnell sich die Teilchen auf der „Jetzt-Ebene“ bewegen.

    Ich meinte ja nur, dass wenn sich alles mit c bewegt, dann passt es recht gut mit der Aussage zusammen:

    Alles bewegt sich „in der Summe“ mit Lichtgeschwindigkeit.
    (Epstein).

    Aber im Grunde genommen kann man jede beliebige „Geschwindigkeit“ im “jetzt” besitzen, man muss nur auf der Ebene bleiben.
    ————————————
    Ich sollte nicht immer über das Ziel hinausschießen. Ich wollte ein Modell darstellen, dass es MIR erlaubt, die Zeit aus meinem Kopf zu bekommen.

    Aber ggf. ist die Einheit „Zeit“ ist in diesem Modell grundsätzlich (auch für euch) unnötig, wenn v = c = 1 (dimensionslos) ist.

    Sie ist für mich auch dann „ersetzt“, wenn v<c ist. Dann geht die Uhr halt langsamer.

    ZU: dann bewegt sich nicht mehr alles mit c.

    Nun Änderungen im Feld schon. Und ein bewegtes Proton ist ja eine Änderung des Feldes.

    Wie man einem Feld oder der Energie hier grundsätzlich eine Größe c=1 verpassen kann?

    Keine Ahnung. Ich gebe der „ungebundenen“ Energie jeden Falls ein c=1. Dem „ungebundenen“ Feld auch.

    „Gebunden“, als Bindungsenergie/ Masse/… gehört es wie die Quarks zum „Gesamtobjekt“ …z.B. Proton.

    Und wenn es nur in der Summe aller Amplituden stimmt.

    Denn auch ein Teilchen oder Welle oder…, dass sich mit über c ausbreitet, verlässt hier die Ebene nicht (Gibt ja sonst nichts :-)). Ein „über c“ wäre einfach ein überdurchschnittlich schnelles Teilchen/Objekt.

    Das Kippen der Zeit um 90° ist das eine, die Interpretation da andere.

    Zu:Was “ruckelt und zuckelt” denn da?
    Wie nahe bekommst du die Ebenen zusammen ? Wie schnell kannst du Jetzt,Jetzt,Jetzt sagen? Und der rel. bewegte Beobachte zu dir? Da holst du Luft und der andere sagt JETZT? Und was ist wenn du jetzt sagst und der andere Luft holt? Ist dann Luftholen und jetzt sagen gleichzeitig oder nur wenn beide jetzt sagen?

    Gruß
    Aveneer

  119. #120 MartinB
    5. Oktober 2012

    @Aveneer
    ich verstehe überhaupt nicht, wie sich irgendwas in der “Jetzt”-Ebene bewegen kann, weil die ja genau einen zeitpunkt beschreibt.

    “Wie nahe bekommst du die Ebenen zusammen”
    Nach momentanem Stand der Physik wohl eine Plack-Zeit.

  120. #121 Aveneer
    5. Oktober 2012

    Hallo Martin,
    Zitat:Nach momentanem Stand der Physik wohl eine Plack-Zeit.
    Aber das ist doch eher ein messtechnisches Problem als ein reales?
    Wenn ich aufgrund physikalischer Einschränkungen nicht feiner differenzieren kann, bedeutet es nicht, dass das Objekt selbst ruckelt?

    Die Plank-Zeit gibt doch “nur” vor, ab wann ich zwei Signale wieder getrennt, einem Objekt zuordnen kann. Das entspricht vielmehr dem Problem der Mikroskopie und der Wellenlänge mit der man messen kann.
    Keiner würde sagen, die Proteine hängen zusammen nur weil sie im Lichtmikroskop optisch nicht getrennt werden können?

    Ansonsten hüpft deine Welt von einem Frame zum anderen Frame. Und zwischen jedem Frame verschwindet das ganze Universum?
    ZU: ich verstehe überhaupt nicht, wie sich irgendwas in der “Jetzt”-Ebene bewegen kann, weil die ja genau einen zeitpunkt beschreibt.

    Mein Bild ist klar anders:
    Also die „Jetzt-Ebene“ bekommen wir alle zunächst hin und die „Zeitpfeile kippen“ doch eigentlich auch? Gedanklich. Und so einfach ist es auch schon. Man achtet nur auf das Falsche.

    Das Ding das an mir vorbeirauscht ist von Ort A bis B wie ich im selben Jetzt”. Immer=stetig.

    Ich achte weniger auf die Bewegung im Raum (der Ebene) sondern die „Existenz“ (hört sich schlimm an, ich weiß es aber nicht besser auszudrücken) die Gegenwart – das Vorhandensein?

    Der Teilchen auf dieser Ebene. Das ist das jetzt. Die „Gegenwart“ (die wortwörtliche „Existenz“) ist ein vom Ort und Abstand unabhängiger Zustand, der sich stetig mit allen anderen Objekten im „jetzt“ befindet.

    Verstehst du den Satz oder klingt das grausam.

    Während der Stein an mir vorbeifliegt, so ist der Zustand des „Vorhandenseins“ ein vom Ort, Abstand und Geschwindigkeit unabhängiger Zustand. Der Zustand des „Vorhandenseins“ ist ein stetiger Zustand ohne jegliches Hüpfen von Frame zu Frame. Auch unterliegt das Vorhandensein selbst wohl auch kaum der Zeitdilatation?

    Da das „Vorhandensein“ (in dieser Ebene) selbst ein Zustand ist der von Ort, Abstand und Impuls unabhängig ist, bilden die Teilchen (unabhängig vom Ort, Abstand und Impuls) eine „Jetzt-Ebene“.

    Gut man könnte es auch die Ebene des „Existieren“ nennen oder „Ebene der Gegenwärtigkeit“ hört sich aber alles noch schlimmer an. Das ist aber die jetzt-Ebene (dort wo alle Teilchen „gerade“ sind-oder eben eigentlich auch immer). Dort wo auch alle “Jetzt” stattfindenden Ereignisse liegen.

    Dass die Teilchen sich auf der Ebene bewegen (mit Ihrem jeweiligen v), macht dem „Vorhandensein“ nichts aus, und fördert nur die Wechselwirkung.

    Die Jetzt-Ebene wird durch die Teilchen gebildet – unabhängig vom Ort, Abstand, Impuls. Auf dieser Ebene bewegen sich die Teilchen (hin und her mit ihrem v bzw. p). Und erzeugen Wechselwirkung. Photonen rauschen der Ebene entlang treffen auf andere. Gleichzeitig bewegen sich die Teilchen von A nach B. Alles andere ist Vergangenheit oder Zukunft (wenn du es nicht auf der Ebene sehen kannst)

    Nicht real und daher physikalisch nicht (mehr) relevant.

    Wenn, dass nicht reicht, dann hilft nur noch das Höhlengleichnis.

  121. #122 Aveneer
    5. Oktober 2012

    Texte ich dich zu? Sag halt!

    Z.B. Relativität der Gleichzeitigkeit:

    Was bedeutet dass? A ich schaue auf meine Uhr der Andere auch… – stimmt was nicht = relativ..
    Passt. Ja die Beispiele passen immer und auch ich erhalte dasselbe Ergebnis wie ihr. Kein Problem.

    Aber ist dass die Gleichzeitigkeit, die man mit dem „Jetzt“ auch meint? I

    Nein, für mich ist gleichzeitig die Tatsache, dass wir gleichzeitig in einem Universum waren und daraus resultiert, dass wir irgendetwas gleichzeitig gemacht haben müssen?

    Es ist doch so:

    Man könnte zu jedem Zeitpunkt „x“ das gesamte Universum anhalten. Je nachdem was bei dem einen oder andern nachher auf der Uhr steht ist egal. Der eine sagt um 12 der andere um 13 und der dritte X Uhr….egal…

    Aber man würde ein Foto erhalten! Auf dem jedes Teilchen zu sehen ist? So und das ist das JETZT.

    Was ist da relativ?

    Es ist dabei völlig unabhängig wer das Foto macht und wann. Das sind alles Schnappschüsse der Jetzt-Ebene. Man muss sich dazu nicht einig sein, wann sie gemacht wurden!

  122. #123 rolak
    5. Oktober 2012

    Um auf Deine Nebenfrage zurückzukommen, Aveneer: ,ittels

    <blockquote>ZuZitierendes</blockquote>

    Plain HTML.

  123. #124 MartinB
    5. Oktober 2012

    @Aveneer
    Ich kann nach wie vor nicht folgen – das liegt vielleicht an deinem Stil, der irgendwie mit Assoziationen zu arbeiten scheint, die ich nicht habe.

  124. #125 Aveneer
    5. Oktober 2012

    @MartinB

    Ich kann nach wie vor nicht folgen – das liegt vielleicht an deinem Stil, der irgendwie mit Assoziationen zu arbeiten scheint, die ich nicht habe.

    Macht nichts. Danke
    @rolak
    Danke

  125. […] ich mich mit dieser Sicht der Dinge sehr gut anfreunden (sie passt auch ganz prima zum Bild des Blockuniversums). Weil hier jede Entscheidung bereits determiniert ist, scheint diese Sicht der Dinge mit der […]

  126. […] doch raumzeitlich betrachtet sind alle "Filmbilder" Teil des "Films" zeit2.jpg QUELLE Zitat von EREIGNISHORIZONT Physikalisch gesehen denke ich, dass Zeit quasi der Grundstoff […]

  127. […] […]

  128. #129 Amy Herzstark
    Berlin
    12. Januar 2016

    Oh man… ich such so verzweifelt nach einem Buch. Unter anderem beschäftigt es sich mit diesem Thema. Das Buch verdeutlicht diese und Themen wie Parallelwelten, Wurmlöcher, Teilchen in Form süßer Kinderbilder mit einem sehr kurzem Text – erst auf den letzten Seiten erfährt man die jeweilige “Theorie” dazu und von wem diese aufgestellt wurde.

    An eine Seite erinnere mich besonders:
    Man sieht einen Stuhl bestehend aus vielen Teilchen. Und der Satz “Beweisen Sie mir, dass dieser Stuhl nicht existiert.” “Welcher Stuhl?”

    Ich weiß weder Verlag, Autor noch Titel… das macht die Suche besonders schwer. Ich habe es vor Jahren verliehen… weiß leider nicht mehr an wen 🙁 Hat jemand eine Ahnung?

  129. #130 MartinB
    12. Januar 2016

    @Amy
    Tut mir Leid, hab ich noch nie von gehört.

  130. #131 rolak
    12. Januar 2016

    Amy Herzstark

    Mist, ein falsch Positiver – der Kommentar wurde heute morgen unbesehen zu ‘gelesen’ entwertet, weil der Name so gut zu denen der Spamwelle(n) passte. Auch der Spamfilter zwischen den Ohren hat so seine Macken ;‑)

    Mir fällt zu der Beschreibung ebenfalls nichts Direktes ein, doch seine Sprache dürfte ja noch bekannt sein und das Erscheinungsdatum läßt sich auch ein wenig eingrenzen. Damit müßte ein wohlinformierter (Fach)Buchhändler sich eigentlich zur Bestellung durchhangeln können.

  131. #132 erik||e oder wie auch immer . . . ..
    Bielefeld
    12. Januar 2016

    @Amy Herzstark
    Habe dein gesuchtes Buch in Bielefeld entdeckt, genauer gesagt: erlebt . . . .. Kann mich noch genau an dein Beispiel mit dem Stuhl erinnern . . . .. Muss dir aber sagen, das dein Buch bisher nicht geschrieben und somit nicht verlegt wurde . . . .. Möchtest du näheres über dein (?) Buch erfahren, dann besuche Bielefeld und nimm dir Zeit den Aluminium-Mann vor der IHK in Bielefeld anzuschauen . . . .. Du findest durch ihn den Eingang in die wundersame Physik der Stadt Bielefeld, die es wohl nicht geben soll . . . ..

    Viel Spaß mit deinem Buch in Bielefeld . . . .. Wenn du willst, treffen wir uns in Bielefeld-NICHT . . . ..

  132. #133 Herr Senf
    12. Januar 2016

    Das Problem haben wir mal vor 40 Jahren durchdiskutiert:
    die Lösung war – ein Stuhl ist kein Tensor!

  133. #134 Feodor
    Reinheim
    14. Februar 2019

    Mit 3 Vorbemerkungen möchte ich zunächst testen, ob Kommentieren noch möglich ist:

    MartinB: “Das ‘Fließen’ der Zeit ist letztlich nur eine Illusion. […Es gibt] kein ‘Vergehen’ der Zeit.”
    Zustimmung. Zeit ist ein abstraktes Konzept, d.h. definitionsgemäß ohne konkreten Bezugsgegenstand. Abstrakta lassen sich nur denken, sie können daher weder fließen noch vergehen. Es ist aber durchaus sinnvoll davon zu sprechen, dass z.B. Wasser fließen kann, auch wenn das nur eine grobe mesokosmische Umschreibung der hierbei stattfindenden Prozesse darstellt. Zeit ist jedoch ein Konzept, das sich bei der Beschreibung des Verlaufs verschiedenster Prozesse außerordentlich bewährt hat.

    MartinB: “Dass es einen Zeitpfeil gibt, ist unbestritten.”
    Da es hier sehr auf Feinheiten ankommt, könnte diese Formulierung vielleicht ein wenig zu großzügig sein. “Zeitpfeil” ist schließlich auch ein Abstraktum. Reale Abstrakta gibt es jedoch nur für Platoniker. Was m.E. eher gemeint sein könnte, ist, dass viele physische Prozesse de facto (wegen Dekohärenz oder Entropiezunahme) irreversibel d.h. gerichtet ablaufen. Experimentell konnte bislang schließlich auch noch niemand ein (hinreichend) geschlossenes System vorweisen, in dem die Entropie tatsächlich abnimmt. Irreversible Prozesse geben offenbar die Richtung des Weltgeschehens an, wohl zumindest in dem Ausschnitt des Universums der uns messtechnisch zugänglich ist.

    MartinB: “Damit der ‘Jetzt-Pfeil’ eine Geschwindigkeit haben kann, bräuchten wir eine zweite, andere Zeit, in der wir die Geschwindigkeit messen könnten. Die gibt es in unserer Welt allerdings nicht.”
    Unter einem Prozess verstehe ich die sequenzielle Zustandsänderungen physischer Objekte. Wahrnehmbare und messbare Prozesse sind zugänglich. Einige dieser Prozesse, die besonders gleichmäßig durch sog. Uhren hervorgebracht werden, dienen als Referenzprozesse, um sie mit anderen Prozessen vergleichen zu können. Zeitmessung (in Eigenzeit) bedeutet also schlichtes Abzählen von Zustandsänderungen einer prinzipiell beliebigen, gleichmäßig arbeitenden Apparatur. Was die Genauigkeit angeht, sind 2 Aspekte wesentlich: Eine Apparatur, die im parallelen Vergleich zu einer anderen mehr Zustandsänderungen durchläuft, kann wegen der feineren Granularität genauer sein. Zusätzlich kommt es noch auf die Regelmäßigkeit der Zustandsänderungen an, die sich etwa durch den Vergleich mehrerer parallel arbeitender Apparate vom gleiche Typ (etwa über Schwebungen) abschätzen lässt. Kurz: Zeit ist ein prozessrelationaler Begriff. Es ist nicht zu sehen, warum eine absolute Eigenzeit benötigt wird.

    Fehlerhinweise nehme ich gerne entgegen.

  134. #135 Feodor
    Reinheim
    16. Februar 2019

    Leider sind im letzten Posting einige Schreibfehler stehen geblieben. 🙁 Ich trage auch noch den Gedanken nach, dass es sinnvoll ist, die Geschwindigkeiten realer physischer Prozesse, die sich messtechnisch vergleichen lassen, von rein fiktionalen und daher beliebigen Zeitvorstellungen zu trennen.

    Bevor ich fortfahre, möchte ich noch darauf hinweisen, dass ich in vieler Hinsicht mit Martin Bäker übereinstimme. Seine Einlassung, “Bewusstsein ist ein Phänomen, das durch den Körper erzeugt wird, genauso wie z.B. ‘Bewegung'”, könnte auch von mir stammen.

    Die Vorstellung des Blockuniversums wird oben ausgehend von einer Betrachtung der Begriffe “jetzt” und “hier” entwickelt. Diese beiden Ausdrücke sind Begriffe der Eigenzuschreibung eines Standpunktes durch ein autonomes, denkfähiges System. Es handelt sich hierbei um einen Referenzpunkt, dessen Zeit- und Raumparameter (oder relativitätstheoretisch: dessen Koordinate im Minkowski-Raum) an das physische System gekoppelt ist, welches die Eigenzuschreibung vornimmt. “Jetzt” bedeutet, gedanklich die Eigenzeit t=0 zu setzen, während “jetzt” gedacht wird. “Hier” legt den gedachten Raumkoordinatenursprung in das autonome System.

    Typischerweise werden die genannten autonom denkfähigen Systeme auf der Erde mit den Menschen identifiziert. Die menschliche Wahrnehmung von Gegenwart, d.h. des Zeitabschnitts, der jeweils als Gegenwart empfunden wird ist ca. 3 Sekunden lang. Darauf folgt, dass das menschliche Zeitempfinden recht ungenau ist. Der Begriff “jetzt” lässt sich aber auch verallgemeinernd auf Messapparate anwenden. “Jetzt” entspricht dabei einem existenten internen Apparatezustand, der (aufgrund der Signallaufzeit) weder durch einen anderen Apparat detektiert noch von einem Beobachter wahrgenommen werden kann. (Hinweis: Quantenverschränkung ermöglicht keine überlichtschnelle Informationsübertragung.)

    Der (Eigen)Zeit-Begriff ist hier wieder über Zustandsänderungssequenzen und Prozessvergleich (Uhrenprozess vs untersuchter Prozess) abgeleitet. Es wird unterstellt, dass Wechselwirkungseffekte aufgrund veränderter Zustände generell erst auftreten, nachdem sich ein Zustand geändert hat (Kausalität). (Verschränkte Quanten bleiben bis zur Dekohärenz schlicht immer korreliert. Bislang keine Informationsübertragung durch Verschränkung.)

    Der von mir erläuterte eigenzeitliche Gegenwartsbegriff baut nicht auf der Vorstellung einer “fließenden” oder “vergehenden” Zeit auf und ist daher mit der Feststellung “die Zeit fließt und vergeht nicht” kompatibel. Benutzt wird stattdessen der Prozessbegriff und die Feststellung einer verzögerten (mit Lichtgeschwindigkeit stattfindenden) Signalausbreitung. Ich halte die “fließende Zeit” nur für eine Metapher, die andeutet, dass sich die Zustände der Dinge in der Welt laufend ändern. Wörtlich interpretiert ist diese Metapher falsch.

    MartinB: “Die Idee, dass es kein ‘Vergehen’ der Zeit gibt, bezeichnet man auch als ‘Blockuniversum’.” Damit müsste auch ich das Konzept des Blockuniversums vertreten, was aber nicht zutrifft. Ich stimme selbstverständlich aber zu, dass es bei Gültigkeit der Relativitätstheorie (vor der ich ausgehe), kein universelles Konzept von Gleichzeitigkeit geben kann. Insofern nehme ich an, dass es zwar eine Gleichzeitigkeitsillusion gibt, eine generelle eigenzeitliche Gegenwartsillusion aber eher unplausibel ist. Für das Konzept eigenzeitlicher Veränderung physischer Objekte wird nämlich überhaupt keine absolute Gleichzeitigkeit benötigt. Die Relativitätstheorie zerstört meines Wissens im übrigen auch nicht die Kausalordnung.

    Ich stimme zu, dass die im Wikipedia-Artikel John Lucas zugeschriebenen Sätze der Kritik am Konzept des Blockuniversums unzureichend sind. Des weiteren ist die Willensfreiheit ohnehin ein inkohärentes Konzept, so dass auch damit nicht gegen die Vorstellung eines Blockuniversums argumentiert werden kann. Schließlich kann man aber auch ohne Dualismus von Gegenwart sprechen, wenn man sich auf den Prozessbegriff bezieht.

    Martin Bäker drückt sich sehr vorsichtig aus. Da ist zwar ein charmanter Zug, macht es jedoch schwer aus dem obigen Text herauszulesen, was denn – abgesehen vom Fehlen einer eigenzeitlichen (lokalen) Gegenwart – die “Features” des Blockuniversums sein sollen, die es anderen Konzepten überlegen macht.

    Bei Wikipedia steht dazu: “Blockuniversum – auch Blockzeit, Eternalismus – bezeichnet [… ist die] Vorstellung […, dass] die Gesamtheit der Zeit, also Vergangenheit, Gegenwart und Zukunft, als gleichermaßen gegeben und real aufgefasst [wird]”. An anderer Stelle: “Der Ausdruck Eternalismus bezieht sich auf Positionen, die eine Ewigkeit, d.h. ein Nichtvergehen oder einen Nichtwandel in der Zeit, bestimmter Objekte oder Strukturen postulieren.”

    Was könnte das bedeuten? Dass es keine Prozesse, d.h. keine Veränderung gibt? Dass etwa die Sonne unwandelbar ist? Oder dass die Sonne sowohl als unauffälliger Hauptreihenstern wie auch als roter Riese wie auch als weißer Zwerg präsent ist? Dass meine Wenigkeit sowohl als lebender Mensch als auch als totes Gerippe existiert? Dass der Wärmetod des Universums bereits stattgefunden hat? (Wikipedia dazu: “future events are ‘already there'”) Brian Greene ist auch wenig vorsichtig. Er fasst Zeit als “Eisblock” auf, der “auf ewig an seinem Platz festgefroren” sei. Das liest sich so, als ob es im Universum keine Veränderungen gebe. Zeit kann aber ohnehin nicht festfrieren, das kann nur Materie. Ist es wirklich zwingend oder auch nur plausibel anzunehmen, dass alle Ereignisse bereits geschehen sind?

    Das Unbehagen bei dem Konzept des Blockuniversums kommt vor allem daher, dass es mit der brutal unplausiblen Vorstellung eines statisches Universum einher zu gehen scheint, d.h. eines Universums, in dem es weder deterministische noch zufällige Veränderung gibt. Das statische Universum erscheint daher so plausibel wie das Gehirn-im-Tank-Szenario.

  135. #136 Claus Müller
    Mittelfranken
    1. Mai 2021

    Einst fragte ein Mathematik Professor den Erstsemestler:
    “Woher wissen Sie eigentlich, dass eins plus eins gleich zwei ergibt?”
    Er antwortete:
    “Weil ich es in der Schule einmal so gelernt habe. Beweisen kann ich es aber nicht.”
    Der Professor war verblüfft, dachte kurz nach und stellte keine weiteren Fragen mehr…

    Gedankenexperiment zu Raum und Zeit:
    Angenommen, es gäbe ein fiktives beliebiges in Raum und Zeit unendlich ausgedehntes Universum.
    Frage:
    Wieviel Zeit würden dann dort z.B. zwei beliebig intelligente Wesen benötigen, um die Elemente der Menge der Natürlichen Zahlen N in aufsteigender Reihenfolge hinzuschreiben und gleichzeitig zusammen auch aufsteigend die Elemente der Teilmenge der Primzahlen P?
    (Zugelassene Hilfsmittel unbeschränkt)
    Einzige Voraussetzung:
    Es soll keine Rolle spielen, wann und in welcher Zeitspanne damit begonnen wird.
    Behauptung (Antwort):
    Gleich lange und unendlich lange!
    (Hypothetisches Theorem oder Axiom)

  136. #137 MartinB
    2. Mai 2021

    @Claus
    Ich habe keine Idee, was du mir damit sagen willst oder was das mit dem Artikel zu tun hat.
    Ja, die Menge der ganzen Zahlen und die der Primzahlen haben dieselbe Cantorsche Mächtigkeit, das ist lange bekannt und führt zu lustigen Konsequenzen:
    https://de.wikipedia.org/wiki/Hilberts_Hotel

  137. #138 Claus Müller
    Mittekfranken
    5. Mai 2021

    Wir wissen und beobachten, dass die Gesetze der Thermodynamik (Hauptsätze eins, zwei, drei und?) und die Gesetze der Quantenmechanik (z.B. Doppelspaltexperiment, sog. “Spukhafte Fernwirkung” usw.) den Mikro- und Makrokosmos richtig beschreiben.

    Aus der derzeit beobachtbaren Expansion des Universums folgt aus der Allgemeinen Relativitätstheorie, dass das Universum mit einem sog. Urknall als Anfangsursache begonnen haben muss, ohne zu wissen, wie es sich in fenster Zukunft weiter entwickeln wird. Das hängt vermutlich von der weiteren Entwicklung der Dunklen Energie und der Dunklen Materie und deren Wechselwirkungen untereinander und mit dem sichtbaren und beobachtbaren Universum ab.

    Angenommen ein Forscher beobachtet und misst in fernster Zukunft “mittlerweile” ein “wieder” schrumpfendes Universum, dann würde er mit derselben Logik ein zeitlich und räumlich unendlich ausgedehntes Universum als Anfangsursache postulieren.

    Wie ist das möglich? Beide Annahmen können doch nicht zugleich richtig und falsch sein?!

    Ist es möglich, dass die Mathematik oder unsere logischen Denkstrukturen nicht widerspruchsfrei sind?!

    Gruß Claus
    Elektroingenieur

  138. #139 MartinB
    5. Mai 2021

    @Claus
    “dann würde er mit derselben Logik ein zeitlich und räumlich unendlich ausgedehntes Universum als Anfangsursache postulieren.”
    Nein. Nehmen wir mal an, die Materiedichte wäre so hoch, dass das Universum tatsächlich erst wächst und dann wieder schrumpft. Dann würde der hypothetische Forscher beim Rückrechnen der Gleichungen auch korrekterweise feststellen, dass die Anfangsbedingung eben nicht ein unendliches Universum ist. Da wird ja quantitativ gerechnet (Friedmann-Gleichungen), nicht bloß intuitiv geguckt.

  139. #140 Claus Müller
    Mittelfranken
    6. Mai 2021

    Okay… jetzt nur nochmal nachgefragt:
    Das heißt, unser “hypothetischer Forscher” käme letztlich wieder zwingend oder zwangsläufig auf den Urknall mit einer Art “Anfangssingularität” als Anfangsursache oder Randbedingung für den Beginn des von ihm beobachteten Universums?

  140. #141 MartinB
    6. Mai 2021

    Wenn die Beobachtungsdaten dazu ausreichen, ja. Die zugrundeliegenden Gelichungen sind gewöhnliche Differentialgleichungen 2. Ordnung, da kannst du als Ingenieur doch ohne Probleme aus bekannten Geschwindigkeiten jetzt und zu ein paar früheren Zeitpunkten die Parameter extrahieren. (Analogie: Wenn sich eine Feder jetzt zusammenzieht, musst du daraus auch nicht schließen, dass sie früher mal unendlich war, sondern kannst mit genügend Beobachtungsdaten zurückrechnen und vielleicht sogar sehen, dass sie mal noch kürzer war, dann die Maximallänge hatte und jetzt wieder schrumpft. Ist hier mathematisch dasselbe.)
    Wenn du Details über die zugrundeliegende Logik wissen willst, empfehle ich mal ganz unbescheiden mein Buch zur Relativitätstheorie…

  141. #142 Matthias U
    8. Oktober 2022

    #1

    > “Wir sind jetzt im jetzt!”

    Das Video ist leider futsch.